Download as pdf or txt
Download as pdf or txt
You are on page 1of 50

3 Classification of Elements and

Periodicity in Properties

Past Years NEET Trend

4
No. of MCQs

0
2021 2020 2019 2018 2017 2016 2015 2014 2013 2012

Investigation Report
TARGET EXAM PREDICTED NO. OF MCQs CRITICAL CONCEPTS
• Acid base behaviour of oxides and hydroxides,
NEET 0-1 Periodic trends in physical properties of element

Perfect Practice Plan


Topicwise Questions Learning Plus Multiconcept MCQs NEET Past 10 Years Total MCQs
Questions
51 26 19 11 107
GENESIS OF PERIODIC CLASSIFICATION properties should occupied the same vertical columns and leaving
out blank spaces where necessary.
(a) Dobereiner’s Triads
He arranged similar elements in the groups of three elements This table was divided into nine vertical columns called groups
known called as triads, in which the atomic mass of the central and seven horizontal rows called periods.
element was merely the arithmetic mean of atomic weight of Periods Number of Elements Called as
other two elements or all the three elements possessed nearly the
same atomic weight. (1)st n = 1 2 Very short period
Dobereiner’s Triads (2)nd n = 2 8 Short period
Li Na K (3)rd n = 3 8 Short period
7 23 39 (7 + 39)/2 = 23
(4)th n = 4 18 Long period
Fe Co Ni
(5)th n = 5 18 Long period
55.85 58.93 58.71 Nearly same atomic masses
(6)th n = 6 32 Very long period
It was restricted to few elements, hence discarded.
(7)th n=7 19 Incomplete period
(b) Newland’s Law of Octave
The groups were numbered as I, II, III, IV, V, VI, VII, VIII and
He was the first to correlate the chemical properties of the elements Zero group.
with their atomic masses.
● Mendeleev’s predicted the properties of those missing
According to octave, if the elements are arranged in the order of elements from the known properties of the other elements in
their increasing atomic weight the eighth element starting from
the same group.
given one is similar in properties to the first element.
● Eka - aluminium and Eka-silicon names were given
This arrangement of elements is called as Newland’s law of for gallium and germanium (not discovered at the time
Octave.
of Mendeleev’s). Later on it was found that properties
Newlands's Octaves predicted by Mendeleev’s for these elements and those found
Element Li Be B C N O F experimentally were almost similar.
At. wt. 7 9 11 12 14 16 19
Eka-alu- Eka-sili- Germa-
Gallium
Element Na Mg Al Si P S Cl Property minium con (pre- nium
(found)
At. wt. 23 24 27 29 31 32 35.5 (predicted) dicted) (found)

Element K Ca Atomic Mass 68 70 72 72.6


At. wt. 39 40
Density/(g/cm3) 5.9 5.94 5.5 5.36
This classification worked quite well for the lighter elements but it
Melting point (K) Low 30.2 High 1231
failed in case of heavier elements and therefore, it was discarded.
Formula of oxide E2O3 Ga2O3 EO2 GeO2
(c) Lothar Meyer’s Classification
He determined the atomic volumes by dividing atomic masses Formula of chloride ECl3 GaCl3 ECl4 GeCl4
with their densities in solid states. Table: Mendeleev's Predictions for the Elements Eka-aluminium
He plotted a graph between atomic masses against their respective (Gallium) and Eka-silicon (Germanium)
atomic volumes for a number of elements. He found the following
observations. Merits of Mendeleev’s Periodic table
Elements with similar properties occupied similar positions on He has simplified and systematised the study of elements and
the curve. their compounds.
He has helped in predicting the discovery of new elements on the
On the basis of his observations he concluded that the atomic
volumes (a physical property) of the elements are the periodic basis of the blank spaces given in its periodic table.
functions of their atomic weights.
It was discarded as it lacks practical utility . Demerits in Mendeleev’s Periodic Table
● Position of hydrogen is uncertain. It has been placed in IA
(d) Mendeleev’s Periodic Table and VIIA groups because of its resemblance with both the
According to him the physical and chemical properties of the groups.
elements are the periodic functions of their atomic masses. ● No separate positions were given to isotopes.
He arranged the known elements in order of their increasing ● Anomalous positions of lanthanides and actinides in periodic
atomic masses considering the facts that elements with similar table.
50 Dropper NEET
MODERN PERIODIC LAW AND PRESENT FORM ● The names are derived by using roots for the three digits in
OF PERIODIC TABLE the atomic number of the element and adding “ium” at the
end. The roots for the numbers are.
Moseley studied (1909) the frequency of the X-ray produced by
Table: Notation for IUPAC Nomenclature of Elements
the bombardment of a strong beam of electrons on metal target.
He found that the square root of the frequency of X-rays ( )
v Digit
0
Latin Word
nil
Abbreviation
n
is directly proportional to number of effective nuclear charge (Z)
of metal i.e. to atomic number and not to atomic mass of the atom 1 un u
of that metal (as nuclear charge of metal atom is equal to atomic 2 bi b
number), i.e. ( v ) = a (Z - b). 3
4
tri
quad
t
q
Where ‘a’ and ‘b’ are constant. Thus, he, concluded that atomic 5 pent p
number was a better fundamental property of an element than its 6 hex h
atomic weight. Then he suggested that the atomic number (Z) 7 sept s
instead of atomic weight should be basis of the classification
8 oct o
of the elements.
9 enn e
Modern Periodic Law (Moseley’s Periodic Law) Table: Name and Symbols in current Use (or proposed) for
Physical and chemical properties of the elements are the periodic Trans-fermium Elements (Z=101-118)
functions of their atomic number.
Atomic number Systematic 1977 IUPAC 1997
If the elements are arranged in order of their increasing atomic
101 Unnilunium (Unu) Mendelevium (Md)
number, after a regular interval, elements with similar properties are
repeated. 102 Unnilbium (Unb) Nobelium(No)
103 Unniltrium(Unt) Lawrencium(Lr)
Periodicity
104 Unnilquadium(Unq) Rutherfordium(Rf)
The repetition of the properties of elements after regular intervals
105 Unnipentium(Unp) Dubnium(Db)
when the elements are arranged in the order of increasing atomic
number is called periodicity. 106 Unnilhexium (Unh) Seaborgium(Sg)
The periodic repetition of the properties of the elements is due to 107 Unnilseptium(Uns) Bohrium(Bh)
the recurrence of similar valence shell electronic configurations 108 Unniloctium(Uno) Hassium (Hs)
after certain regular intervals. For example, alkali metals have 109 Unnilennium(Une) Meitnerium(Mt)
same valence shell electronic configuration ns1 , therefore, have
110 Ununnillium(Uun) Darmstadtium(Ds)
similar properties.
111 Unununium(Uuu) Rontgenium(Rt)
The modern periodic table consists of horizontal rows (periods)
and vertical column (groups). 112 Ununbium(Uub) Copernicium (Cn)
113 Ununtrium(Uub) Nihonium(Nn)
Periods
114 Ununquadium(Uuq) Flerovium (Fl)
There are seven periods numbered as 1, 2, 3, 4, 5, 6 and 7.
115 Ununpentium(Uup) Moscovium(Mc)
● Each period consists of a series of elements having same
116 Ununhexium(Uuh) Livermorium (Lv)
valence shell.
117 Ununseptium(Uus) Tennessine(Ts)
● Each period corresponds to a particular principal quantum
number of the valence shell present in it. 118 Ununoctium(Uuo) Oganesson(Og)
● Each period starts with an alkali metal having outermost CLASSIFICATION OF THE ELEMENTS
electronic configuration as ns1.
It is based on the type of orbitals which receives the differentiating
Groups electron (i.e., last electron).
(a) s-block elements: When shells upto (n – 1) are completely
● There are eighteen groups numbered as 1, 2, 3, 4, 5, ........... filled and the last electron enters the s-orbital of the outermost
13, 14, 15, 16, 17, 18. (nth) shell, the elements of this class are called s-block elements.
● Group consists of a series of elements having similar valence ● Group 1 & 2 elements constitute the s-block.
shell electronic configuration. ● General electronic configuration is ns1-2
● s-block elements lie on the extreme left of the periodic table.
IUPAC NOMENCLATURE FOR ELEMENTS ● Includes metals.
WITH Z >100 (b) p-block elements: When shells upto (n – 1) are completely
● Nomenclature of elements CNIC (commission on nomenclature filled and last electron enters the p-orbital of the nth orbit, elements
of inorganic chemistry) appointed by IUPAC in 1994, approved of this class are called p-block elements.
a nomenclature scheme as well as also gave official names ● Group 13 to 18 elements constitute the p-block.

for elements after Z > 100 (upto atomic number 104 to 109 ● General electronic configuration is ns2 np1-6
discovered by that time). This nomenclature is to be followed for ● p-block elements lie on the extreme right of the periodic
naming the elements until their names are officially recognised. table.
Classification of Elements ... 51
● This block includes some metals, all non metals and TRAIN YOUR BRAIN
metalloids. Q. Elements A, B, C, D and E have the following electronic
● s-block and p-block elements are collectively called main configurations :
group or representative elements. A : 1s2 2s2 2p1 B : 1s2 2s2 2p6 3s2 3p1
2 2
C : 1s 2s 2p 3s 3p6 2 3 D : 1s2 2s2 2p6 3s2 3p5
(c) d-Block elements
2 2 6
E : 1s 2s 2p 3s 3p 4s2 6 2
When outermost (nth) and penultimate shells (n–1)th shells
are incompletely filled and last electron enters the (n–1) d Which among these will belong to the same group in the
orbitals (i.e., d-orbital of penultimate shell) then elements of periodic table ?
this class are called d-block elements. Ans. Out of these, elements A and B will belong to the same group
● Group 3 to 12 elements constitute the d-block. of the periodic table because they have same outer electronic
configuration, ns2 np1.
● General electronic configuration is (n–1) d1-10 ns0-2 (except,
palladium which has valence shell electronic configuration Q. An element X with Z = 112 has been recently discovered.
4d10 5s0). What is the electronic configuration of the element ? To
which group and period will it belong ?
● d-block elements are classified into four series.
Ans. (a) T he electronic configuration of element X is [Rn]86 5f14
● Those elements which have partially filled d-orbitals in neutral
6d107s2
state or in any stable oxidation state are called transition elements.
(b) It belongs to d-block as last electron enters in d subshell.
● All the transition elements are metals and most of them form
(c) As number of electrons in (n – 1)d subshell and valence shell
coloured complexes or ions.
is equal to twelve i.e. 10 + 2. So it belongs to group 12.
(d) f-Block elements (d) It belongs to period 7 of the periodic table as principal
When n, (n–1) and (n–2) shells are incompletely filled and quantum number of valence shell is 7 (i.e., 7s2).
last electron enters into f-orbital of antipenultimate i.e., (n–2)
th shell, elements of this class are called f-block elements. PERIODIC TRENDS IN PROPERTIES OF ELEMENT
● General electronic configuration is (n – 2) f 1-14 (n – 1) d 0-1 Trends in Physical Properties
ns2 (i) Effective Nuclear Charge: In an polyelectronic atom between
● The elements coming after uranium are called transuranium the outer most valence electrons and the nucleus of an atom,
elements. there exists number of shells containing electrons. Due to the
● They are also called as inner-transition elements as they presence of these inner electrons, the valence electrons are unable
contain three outer most shell in complete and were also to experience the attractive pull of the actual number of protons
referred to as rare earth elements since their oxides were in the nucleus. These inner electrons act as shield between the
rare in earlier days. valence electrons and protons in the nucleus.
The elements of f-blocks have been classified into two series. Thus, the presence of intervening (shielding) electrons reduces
the electrostatic attraction between the protons in the nucleus
1. Ist inner transition or 4 f-series, contains 14 elements58Ce and the valence electrons because intervening electrons repel the
to 71Lu. Filling of electrons takes place in 4f subshell.
valence electrons.
2. IInd inner transition or 5 f-series, contains 14 elements
The effective nuclear charge (Zeff) is the charge actually felt by
90Th to 103Lr. Filling of electrons takes place in 5f subshell.
the valence electron. Zeff is given by Zeff = Z – σ, (where Z is the
● The actinides and lanthanides have been placed at the bottom actual nuclear charge (atomic number of the element) and σ is the
of the periodic table to avoid the undue expansion of the shielding (screening) constant).
periodic table.
(ii) Atomic radius: Atomic radius is taken as the effective size
 KEY NOTE that is the distance of the closest approach of one atom to another
Prediction of period, group and block atom in a given bonding state.
Š Period of an element corresponds to the principal quantum
number of the valence shell.
Types of Atomic Radius
Š The block of an element corresponds to the type of subshell (a) Covalent radius : It is one-half of the distance between the
which receives the last electron. centres of two nuclei (of like atoms) bonded by a single covalent
Š The group is predicted from the number of electrons in the bond as shown in figure.
valence shell or/and penultimate shell as follows.
(a) For s-block elements, Group number = the number of
valence electrons
(b) For p-block elements, Group number = 10 + number
of valence electrons
(c) For d-block elements, Group number = number of
electrons in (n – 1) d sub shell + number of electrons
in valence shell.
Covalent radius is generally used for non metals.

52 Dropper NEET
Single Bond Covalent Radius, SBCR (bond length)
Variation in a Period Variation in a Group
(a) For homodiatomic molecules dA–A = rA + rA or 2rA
d A− A As a result, the electrons are The effect of increased number
so, rA =
pulled closer to the nucleus by of atomic shells overweights
2
(b) For heterodiatomic molecules in which electronegativity 1 the effect of increased nuclear
the increased Zeff. rn ∝ charge. As a result of this the size
remains approximately same. Z* of atom increases from top to
dA – B = rA + rB Hence atomic radii decrease with bottom in a given group.
For heteronuclear diatomic molecule, A–B, where difference increase in atomic number in a
between the electronegativity values of atom A and atom B is period from left to right.
relatively larger,
dA – B = rA + rB – 0.09 Δχ The atomic radius of inert gas (zero group) is given largest in
Electronegativity values are given in Pauling units and radius in a period because it is represented by vander Waals’s radius is
picometers. generally larger than the covalent radius.
Δχ = XA – XB where XA and XB are electronegativity values The Van der Waal’s radius of inert gases also increases from top
of high electronegative element A and less electronegative to bottom in a group.
element B. In the transition series (e.g. in first transition series), the covalent
Van der Waal’s radius (Collision radius) : It is one-half of the radii of the elements decrease from left to right across a row until
internuclear distance between two adjacent atoms in two nearest near the end when the size increases slightly.
neighbouring molecules of the substance in solid state as shown
in figure. The radii of the elements from Cr to Cu, are very close to one
another as the successive addition of d-electrons screen the outer
electrons (4s) from the inward pull of the nucleus. As a result of
this, the size of the atom does not change much in moving from
Cr to Cu.
There are 14 lanthanide elements between lanthanum and
hafnium, in which the antipenultimate 4f shell of electrons (exert
very poor shielding effect) is filled. There is a gradual decrease in
size of the 14 lanthanide elements from cerium to lutetium. This
(c) Metallic radius (Crystal radius): It is one-half distance is called lanthanide contraction. This lanthanide contraction
between the nuclei of two adjacent metal atoms in the metallic
cancels out the normal size increase on descending a group in
crystal lattice as shown in figure.
case of transition elements.
(iii) Ionic radius: It is the effective distance from the centre of
nucleus of the ion that is cation /anion up to which it has an
influence in the ionic bond is know ionic radius.
Cl– (ionic radius Cl (covalent
1.84 Å) radius 0.99 Å)
Number of electrons : 17 18
Number of protons : 17 17
Thus, the covalent, Vander Waal’s and metallic radius magnitude
wise follows the order, So, there is reduction in effective nuclear charge and hence the
rcovalent < rcrystal < rvander waal's electron cloud expands in case of Cl–.
The sizes of ions increases as we go down a group (considering
Table: Atom Radius Variation the ions of same charge). For example :
Li+ (0.76) < Na+ (1.02) < K+ (1.38) < Rb+ → (in Å)
Variation in a Period Variation in a Group
Be2+ < Mg2+ < Ca2+ < Sr2+
In a period left to right In a group top to bottom
F– < Cl– < Br– < I–
Nuclear charge (Z) increases by Nuclear charge (Z) increases by The species containing the same number of electrons but differ in
one unit more than one unit the magnitude of their nuclear charges are called as isoelectronic
Effective nuclear charge (Zeff) Effective nuclear charge (Zeff) species. For example, N3– , O2–, F–, Ne, Na+ , Mg2+ and Al3+ are
also increases almost remains constant because all isoelectronic species with same number of electrons (i.e 10)
of increased screening effect of but different nuclear charges of +7, +8, +9, +10, +11, +12 and +13
inner shells electrons. respectively.
But number of orbitals (n) But number of orbitals (n) Within a series of isoelectronic species as the nuclear charge
remains constant increases. increases, the force of attraction by the nucleus on the electrons
also increases. So, the ionic radii of isoelectronic species decrease
Classification of Elements ... 53
with increases in the magnitude of nuclear charges. For example, (b) Nuclear Charge: The ionisation energy increases with
as shown in figure increase in the nuclear charge.
It is due to the fact that with increase in the nuclear charge,
the electrons of the outermost shell are tightly held by the
nucleus and hence large amount of energy is required to pull
out an electron from the atom.
As effective nuclear charge decrease. Ionisation energy increases as we move from left to right
● Following are the examples of isoelectronic series
along a period due to increase in nuclear charge.
(i) S2–, Cl–, K+ , Ca+2 , Sc+3 (ii) SO2, NO3– , CO32–,
(c) Shielding or screening effect: The electrons in the innermost
(iii) N2 , CO, CN– (iv) NH3 , H3O+
shells will act as a screen or shield between the nucleus and
● Pauling’s empirical formula for ionic radius
the electrons in the outermost shell. This is called shielding
1 effect. The larger the number of electrons in the inner shells,
∝ (only for isoelectronic species)
nuclear ch arg e greater is the screening effect and smaller the force of
attraction and thus ionization energy (IE) decreases.
TRAIN YOUR BRAIN Screening Effect
Q. X – X bond length is 1.00 Å and C–C bond length is 1.54 Å.
If electronegativities of X and C are 3.0 and 2.0 respectively,
then C–X bond length is likely to be ? (using Stevension &
Schomaker formula).
Ans. rC–X = rC + rX – 0.09 Δχ
1.00 1.54 (d) Penetration effect of the electron: The ionization energy also
=+ − 0.09 [ ∆χ= 1=
] 1.27 − 0.09 depends on the type of electron that is removed. s, p, d and
2 2 f electrons have orbitals with different shapes. An s electron
C–X bond length = 1. 18 Å. penetrates closer to the nucleus, and is therefore more tightly
Q. Mg2+ is smaller than O2– in size, though both have same held than a p electron. Similarly p-orbital electron is more
electronic configuration. Explain ? tightly held than a d-orbital electron and a d-orbital electron
Ans. Mg2+ and O2– both are isoelectronic i.e., have same number is more tightly held than an f-orbital electron.
of electrons. But Mg2+ having 12 protons in its nucleus apply If other factors being equal, penetration order are in the order
s > p > d > f.
a higher effective nuclear charge than O2– having 8 protons
and thus valence shell as well as inner shells electrons are (e) Electronic Configuration: If an atom has exactly half-filled
more strongly attracted by the nucleus in Mg2+ resulting or completely filled orbitals, then such an arrangement has
smaller size than O2–. extra stability.
(iv) Ionisation Energy (IE): Ionisation energy (IE) is defined as The removal of an electron from such an atom requires more
the amount of energy required to remove the most loosely bound energy then expected.
electron from an isolated gaseous atom to form a cation. (v) Electron Affinity: The electron gain enthalpy (ΔegH) is the
change in standard molar enthalpy when a neutral gaseous atom
gains an electron to form an anion.
M ( g )  → M + ( g ) + e − : M + ( g ) + IE2 → M 2 + ( g ) + e −
1 E

X (g) + e– (g) → X– (g)


M2+ (g) + IE3 → M+3 (g) + e– Electron gain enthalpy provides a measure of the ease with which
IE1, IE2 & IE3 are the Ist, IInd & IIIrd ionization energies to an atom adds an electron to form anion. Electron gain may be
remove electron from a neutral atom, monovalent and divalent either exothermic or endothermic depending on the elements.
cations respectively.
When an electron is added to the atom and the energy is released,
In general, (IE)1 < (IE)2 < (IE)3 < .............., as the number of the electron gain enthalpy is negative and when energy is needed
electrons decreases, the attraction between the nucleus and the to add an electron to the atom, the electron gain enthalpy is
remaining electrons increases considerably and thus subsequent positive.
ionization energies increase.
Ea = E (X, g) – E (X–, g),
● It is measured in kJ mol–1, k Cal mol–1, eV (electron volt).
An element has a high electron affinity if the additional electron can
● Factors Influencing Ionisation energy
enter a shell where it experiences a strong effective nuclear charge.
It is influenced by the following factors.
Across a period, with increase in atomic number, electron gain
(a) Size of the Atom: Ionisation energy decreases with increase
enthalpy becomes more negative because left to right across a
in atomic size.
period effective nuclear charge increases and as a result it will be
As the distance between the outermost electrons and the easier to add an electron to a small atom.
nucleus increases, the force of attraction between the valence
shell electrons and the nucleus decreases. As a result, outer As we move in a group from top to bottom, electron gain enthalpy
most electrons are held loosely and lesser amount of energy becomes less negative because the size of the atom increases and
is required to break them. the added electron would be at larger distance from the nucleus.

54 Dropper NEET
The electronegativity also increases as the s-character in the
 KEY NOTE hybrid orbitals increases.
Š Group 17 elements (halogens) have very high negative Hybrid orbital sp3 sp 2 sp
electron gain enthalpies (i.e. high electron affinity) because
s −character 25% 33% 50%
they can attain stable noble gas electronic configuration by →
Electronegativity increases
picking up an electron.
Š Noble gases have large positive electron gain enthalpies Variation of Electronegativity in a Group
because the electron has to enter the next higher energy level
leading to a very unstable electronic configuration. On moving down the groups, Z increases but Zeff almost remains
constant, number of shells (n) increases, rn (atomic radius) increases.
Thus, electronegativity decreases moving down the groups.
Negative electron gain enthalpy of O or F is less than S or Cl. It is
due to the fact that when an electron is added to O or F, the added Variation of Electronegativity in a Period
electron goes to the smaller n = 2 energy state and experiences While moving across a period left to right, Z, Zeff increases & rn
significant repulsion from the other electrons present in this level.
decreases. Thus, electronegativity increases along a period.
In S or Cl, the electron goes to the larger n = 3 energy state and
consequently occupies a larger region of space leading to much Measurement of Electro negativity
less electron-electron repulsion. (a) Pauling’s scale : Linus Pauling developed a method for
Nitrogen has very low electron affinity because there is high calculating relative electronegativities of most elements.
electron repulsion when the incoming electron enters an orbital According to Pauling
that is already half filled.
1 Δ = XA – XB = 0.208 E. A − B − E A − A × EB − B
Electron affinity ∝
Atomic size (b) Mulliken’s scale : Electronegativity χ(chi) will be the
Electron affinity ∝ Effective nuclear charge (zeff) average of the ionisation energy (IE) and the electron affinity
(EA) of an atom (both expressed in electron volts).
1
Electron affinity ∝ .
Screening effect IE + EA
χM =
Stability of half filled and completely filled orbitals of a subshell 2
is comparatively more and the addition of an extra electron to Paulings’s electronegativity χp is related to Mulliken’s
such an system is difficult and thus, the electron affinity value electronegativity χM as given below.
decreases. χP = 1.35 (χM)1/2 – 1.37
Mulliken’s values were about 2.8 times larger than the
TRAIN YOUR BRAIN Pauling’s values.
Q. Consider the elements N, P, O and S and arrange them in
order of increasing negative electron gain enthalpy.
Ans. Order of increasing negative electron gain enthalpy is N < P  KEY NOTE
< O < S. Application of Electronegativity
(vi) Electronegativity: Electronegativity is a measure of the (a) Nomenclature
tendency of an element to attract shared electrons towards itself Name of more electronegative element is written at
in a covalently bonded molecules. the end and ‘ide’ is suffixed to it. The name of less
The magnitude of electronegativity of an element depends upon its electronegative element is written before the name
ionisation potential & electron affinity. Higher ionisation potential of more electronegative element of the formula. For
& electron affinity values indicate higher electronegativity value. example-
When atomic size increases, the distance between nucleus and Correct formula Name
valence shell electrons increases, thus the force of attraction (a) I+ Cl– Iodine chloride
between the nucleus and the valence shell electrons decreases and (b) Partial Ionic Character in Covalent bonds
so, the electronegativity values also decrease.
Partial ionic characters are induced in covalent
With increase in nuclear charge force of attraction between compounds by the difference of electronegativities.
nucleus and the valence shell electrons increases and, hence, Hanney and Smith calculated percentage of ionic
electronegativity value increases character from the difference of electronegativity.
In higher oxidation state, the element has higher magnitude of Percentage of ionic character
positive charge. = 16(XA – XB) + 3.5(XA – XB)2 = 16Δ + 3.5Δ2
So, due to more positive charge on element, it has higher polarising = (0.16Δ + 0.035Δ2) × 100
power. XA is electronegativity of element A (Higher)
Thus, with increase in the oxidation state of element, its XB is electronegativity of element B (Lower)
electronegativity also increases. Δ = XA – X B

Classification of Elements ... 55


TRAIN YOUR BRAIN having completely filled shells have rather positive electron gain
Q. If electronegativity of x be 3.2 and that of y be 2.2, the enthalpy values). This results in high chemical reactivity at the
two extremes and the lowest in the centre.
percentage ionic character of xy is –
So, the maximum chemical reactivity at the extreme left (among
(a) 19.5 (b) 18.5 alkali metals) is represented by the loss of an electron leading to
(c) 9.5 (d) 29.5. the formation of cation and at the extreme right (among halogens)
Ans. (a) ENx – ENy = 3.2 – 2.2 = 1. represented by the gain of an electron forming an anion.
Δ=1 The loss and gain of electron can be related with the reducing and
[Δ = difference of electronegativity values between x and y] oxidizing behaviour of the elements respectively. Therefore, it can
also be directly related to the metallic and nonmetallic character
% ionic character = 16Δ + 3.5Δ2 = 19.5. of elements.
Thus, the metallic character of an element, that is highest at the
extremely left decreases and the nonmetallic character increases
while moving from left to right across the period.
The chemical reactivity of an element will be best understood by
its reactions with oxygen and halogens.
Nature of oxide
Elements on two extremes of a period easily combine with oxygen
to form oxides.
The normal oxide formed by the element on extreme left is the
most basic (e.g. Na2O), whereas that formed by the element on
The periodic trends of elements in the periodic table extreme right is the acidic (e.g. Cl2O7).
Oxides of elements in the centre are amphoteric (e.g. Al2O3,
Periodic Trends in Chemical Properties As2O3) or neutral (e.g., CO, NO, N2O).
1. Periodicity of Valence or Oxidation States: The valence of Amphoteric oxides behave as acidic with bases and as basic with
representative elements is usually (though not necessarily) equal acids, while neutral oxides have no acidic or basic properties.
to the number of electrons in the outermost orbitals and / or equal In general, metallic oxides (O2–), peroxides (O22 –) and super
to eight minus the number of outermost electrons. Frequently oxides (O21 –) are ionic solids.
used for valence. Oxides of IA and IIA dissolve in water forming basic solutions
where as other oxides do not dissolve in water.
Being highest electronegative element, fluorine is given oxidation Na2O + H2O → 2NaOH
state –1. In a group, basic nature of oxides increases or acidic nature
Since there are two fluorine atoms in this molecule, oxygen with decreases.
outer electronic configuration 2s22p4 shares two electrons with In a period the nature of the oxides varies from basic to acidic.
fluorine atoms and thereby exhibits oxidation state +2. Oxide Nature
On the other hand sodium with electronic configuration 3s1 loses Na2O Strongly basic
one electron to oxygen and is given oxidation state +1.
MgO Basic
There are many elements which exhibit variable valence. This is Al2O3 amphoteric
particularly characteristic of transition elements and actinoids. SiO2 Weakly acidic
2. Anomalous properties of Second period Elements: Some P4O10 Acidic
elements of certain groups of 2nd period resemble much in SO3 Acidic
properties with the elements of third period of next group i.e. Cl2O7 Strongly acidic
elements of second and third period are diagonally related in
properties. This phenomenon is known as diagonal relationship. Nature of Hydroxide
For example, the similarity between lithium (the first member of If electronegativity of E and O is larger than H and O in H2O then
group 1) and magnesium (the second element in group 2) is called EOH is basic.
a diagonal relationship. Diagonal relationship also exist between E—O—H + H2O → [EOH2]+ + OH–
other pairs of elements Be and Al, B and Si as shown in figure ; If electronegativities of E and O is less than that of H and O in
Diagonal Relationship H2O then EOH is acidic due to the formation of H3O+.
E—O—H + H2O → H3O+ + EO–
2nd period Li Be B C
3rd period Na Mg Al Si TRAIN YOUR BRAIN
Q. Arrange the following in decreasing basic nature LiOH,
PERIODIC TRENDS AND CHEMICAL
REACTIVITY NaOH, KOH, CsOH.
The ionization enthalpy of the extreme left element in a period Ans. The basic nature of hydroxides of elements of group 1st
is the least and the electron gain enthalpy of the element on the increases on descending the group with increase in size of
extreme right is the highest negative (except noble gases which cation as CsOH > RbOH > KOH > NaOH > LiOH.
56 Dropper NEET
Topicwise Questions

GENESIS OF PERIODIC TABLE, MODERN 11. Electronic configuration of the 4th transitional element is:
PERIODIC LAW & NOMENCLATURE OF (a) 1s2 2s2 2p6 3s2 3p6 4s2 (b) 1s2 2s2 2p6 3s2 3p6 3d4 4s2
ELEMENTS (c) 1s2 2s2 2p6 3s2 3p6 3d4 (d) 1s2 2s2 2p6 3s2 3p6 3d5 4s1
1. The third period of periodic table contains 12. Which of the following ions does not have the configuration
(a) 8 elements (b) 32 elements of Argon?
(c) 3 elements (d) 18 elements (a) Cl– (b) K+
(c) Ca2+ (d) I–
2. The element Californium belongs to the family of
(a) Actinoids series (b) Alkali metals 13. Which of the following species have the same number of
(b) Lanthanoid series (d) Alkali Earth metals electrons in its outermost as well as penultimate shell?
(a) Mg2+ (b) O2–
3. The tenth element in the periodic table resembles the element
with atomic number (c) F– (d) Ca2+
(a) 2 as well as 30 (b) 2 as well as 54 14. Name an element of p-block of the periodic table in which
(c) 8 as well as 18 (d) 8 only last electron goes to the s-orbital of valence shell instead of
p-orbital:
4. Without looking at the periodic table select from each of the
(a) As
following list, the elements belonging to the same group
(b) Ga
(a) Z = 12, 38, 4, 88 (b) Z = 9, 16, 3, 35
(c) No such element is there
(c) Z = 5, 11, 27, 19 (d) Z = 24, 47, 42, 55
(d) He
5. The plot of square root of frequency of X-rays emitted
against atomic number led to the suggestion of which law 15. Find the total no. of d-block elements in the following
(rule)? Atomic numbers?
(a) Mendeleev’s periodic law 48, 28, 70, 100, 55, 45, 34, 36
(b) Modern periodic law (a) 5 (b) 3
(c) Hund's rule (c) 6 (d) 2
(d) Newland's law 16. Which of the following electronic configuration is of
transition elements?
6. The discovery of which of the following group of element
gave death to the Newland law of octave? (a) 1s2 2s2 2p6 3s2 3p6 4s2
(a) Inert gas (b) Alkaline earth metal (b) 1s2 2s2 2p6 3s2 3p6 3d10 4s2 4p1
(c) Rare earth (d) Actinoid series (c) 1s2 2s2 2p6 3s2 3p6 3d10 4s2 4p6
(d) 1s2 2s2 2p6 3s2 3p6 3d2 4s2
7. The long form of periodic table was based on.
(a) Atomic number (b) Atomic mass 17. Which block of the periodic table contains the element with
configuration 1s2 2s2 2p6 3s2 3p6 3d10 4s1?
(c) Atomic volume (d) Effective nuclear charge
(a) s-block (b) p-block
8. What is the name of element with atomic number 105?
(c) d-block (d) f-block
(a) Kurchatovium (b) Dubnium
18. The electronic configuration of gadolinium (atomic number
(c) Nobelium (d) Holmium
64) is:
(a) [Xe] 4f8 5d9 6s2 (b) [Xe] 4f7 5d1 6s2
ELECTRONIC CONFIGURATION
(c) [Xe] 4f3 5d5 6s2 (d) [Xe] 4f6 5d2 6s2
9. The electronic configuration, element of group 18 can be
19. Which of the following will have total number of d-electrons
represented by.
equal to the difference in the number of total p and s
(a) ns2np5 (b) ns1 electrons?
2
(c) ns np 6 (d) (n-1)d8 ns2 (a) He (b) Ne
10. In transition element , the incoming electron occupies (n-1)d (c) Ar (d) Kr
subshell, in preference to.
20. The radii of the F, F­–, O and O2– are in the order:
(a) np-level (b) ns-level
(a) O2– > O > F– > F (b) F– > O2– > F > O
(c) (n-1) p-level (d) (n+1) s-level 2– –
(c) O > F > F > O (d) O2– > F– > O > F
Classification of Elements ... 57
21. Which of the following configuration represent atoms of 32. Which out of the following has the largest size?
element having the highest second ionisation energy? (a) Rb+ (b) Mg2+
(a) 1s2 2s2 2p4 (b) 1s2 2s2 2p6 (c) Li+ (d) Na+
(c) 1s2 2s2 2p6 3s1 (d) 1s2 2s2 2p6 3s2 33. Which of the following units are used frequently for atomic
22. Which of the following ions are paramagnetic in character? radii?
(a) Zn2+ (b) Cu+ (a) Meter (b) Picometers
(c) Ni2+ (d) Ag+ (c) Kilometers (d) Centimeters
23. Which of the following is not correct for isoelectronic ions? 34. Which of the following does not affect the ionisation
(a) They have the same number of electrons around their nuclei potential of the atom?
(b) Higher the atomic number, higher will be positive charge (a) Nuclear charge (b) Stable configuration
in a series of isoelectronic ions of same period
(c) Penetration effect (d) None of these
(c) Isoelectronic ions have same electric charge
35. The correct order of ionic radii for the ions S2–, Cl–, P3-, Ca2+
(d) An isoelectronic series may have both positively and
is
negatively charged ions
(a) Ca2+ > Cl– > S2– > P3– (b) S2– > P3– > Cl– >Ca2+
24. Which of the following is correct?
(c) P3– > S2– < Cl– > Ca2+ (d) Ca2+ < Cl– < S2– < P3–
(a) rionic ∝ Z (b) rionic ∝ Zeff
36. The correct order of electron affinity of halogens is:
1
(c) rionic
2
∝ (d) rionic ∝ Z eff (a) F > Cl > Br > I (b) I > Br > Cl > F
Zeff
(c) Cl > F > Br > I (d) Cl > F > I > Br
25. Pd has exceptional electronic configuration of 4d10 5s0. It
belong to 37. Which of the following pairs show diagonal relationship?
(a) 4th period, group 11 (b) 5th period, group 10 (a) Li, Mg (b) Be, Al
th
(c) 6 period, group 9 (d) 3rd period, group 16 (c) B, Si (d) All of these
26. Atomic number of Ag is 47. In the same group, the atomic 38. Which of the following element is expected to have highest
numbers of elements placed above and below Ag in long electron affinity?
form of periodic table will be (a) 1s2 2s2 2p6 3s2 3p5 (b) 1s2 2s2 2p3
(a) 29, 65 (b) 39, 79 (c) 1s2 2s2 2p4 ­ (d) 1s2 2s2 2p5
(c) 29, 79 (d) 39, 65 39. Which of the following oxide is amphoteric?
(a) Na2O (b) Al2O3
PERIODIC TRENDS AND PROPERTIES OF THE
ELEMENTS (c) SO3 (d) P2O5
40. The correct order of electronegativity of N, O, F and P is.
27. Which of the following isoelectronic ions has lowest
ionisation energy? (a) F > N > P > O (b) F > O > P > N
(a) K+ (b) Ca2+ (c) F > O > N > P (d) N > O > F > P
(c) Cl– (d) S2– 41. Which set of elements have strongest tendency to form
28. The elements which occupy the peaks of ionisation energy anions?
curve are (a) Na, Cl, Al (b) Cu, Ag, Au
(a) Na, K, Rb, Cs (b) Na, Mg, Cl, I (c) Be, F, N (d) F, Cl, Br
(c) Cl, Br, I, F (d) He, Ne, Ar, Kr 42. Outer most configuration of most electronegative element of
29. Which out of the following has the largest ionisation energy? the periodic table is.
(a) 11Na (b) 19K (a) 3s2 3p6 (b) 2s2 2p5 (c) 4s2 4p5 (d) 2s2 2p4
(c) 12Mg (d) 37Rb 43. In which of the following pairs, the ionisation energy of the
30. Which of the process requires largest energy among them? first species is less than that of the second:
(a) Al(g) → Al+(g) + e– (a) N, P (b) Be2+, Be
(b) Al2+(g) → Al3+(g) + e– (c) N, N– (d) S, P
(c) Al+(g) → Al2+(g) + e– 44. Isoelectronic ions are those which have:
(d) All the processes require same amount of energy (a) Same size
31. Which of the following process refers to the ionisation potential? (b) Same ionisation energy
(a) X(s) → X+(g) + e– (b) X(g) + 2e– → X2–(g) (c) Same electronic configuration
(c) X(g) → X+(g) + e– (d) X(g) + e– → X–(g) (d) Same nuclear charge
58 Dropper NEET
45. The correct order of the size of C, N, P, S following the 49. Which is mismatched regarding the position of the element
order: as given below?
(a) N < C < P < S (b) C < N < S < P (a) X(Z = 89) - f block, 6th period
(c) C < N < P < S (d) N < C < S < P (b) Y(Z = 100) - f block, 7th period
46. Which of the following has smallest size? (c) Z(Z = 115) - d block, 7th period
(a) Al3+ (b) Al2+
(d) Both (a) & (c)
+
(c) Al (d) Al
50. Correct order of Ist ionization potential among elements Be,
47. Which one of the following is an incorrect statement?
B, C, N, O is
(a) The ionisation potential of nitrogen is greater than that of
oxygen (a) B < Be < C < O < N
(b) The electron affinity of fluorine is greater than that of chlorine (b) B < Be < C < N < O
(c) The ionization potential of beryllium is greater than that (c) Be < B < C < N < O
of boron
(d) Be < B < C < O < N
(d) The electronegativity of fluorine is greater than that of
chlorine 51. An atom of an element has electronic configuration 2, 8, 1.
48. Identify the correct order of the size of the following Which of the following statement is correct?
(a) Ca2+ < K+ < Ar < Cl– < S2– (a) The valency of element is 7
(b) Ar < Ca2+ < K+ < Cl– < S2– (b) The element exists as a triatomic molecule
(c) Ca2+ < Ar < K+ < Cl– < S2– (c) The element is metalloid
(d) Ca2+ < K+ < Ar < S2– < Cl– (d) The element forms basic oxide

Learning Plus

1. The size of ionic species is correctly given in the order 6. Which of the following is not an actinoid?
(a) Na+ > Mg+2 > Cl+7 > Si4+ (a) Curium (Z = 96) (b) Californium (Z = 98)
(b) Na+ > Mg+2 > Si4+ > Cl+7 (c) Uranium (Z= 92) (d) Terbium (Z= 65)
(c) Cl+7 > Si+4 > Mg+2 > Na+ 7. The order of screening effect of electrons of s, p, d and f
(d) Cl+7 > Na+ > Mg+2 > Si+4 orbitals of a given shell of an atom on its outer shell electrons is:
2. A neutral atom (A) is converted to (A3+) by the following (a) s > p > d > f (b) f > d > p > s
E1 E2 E3 (c) p < d < s > f (d) f > p > s > d
process : A →
− e−
A + →− e−
A 2 + →
− e−
A 3+
The correct order of E1, E2 and E3 energies is 8. The first ionisation enthalpies of Na, Mg, Al and Si are in the
order:
(a) E1 < E2 < E3 (b) E1 > E2 > E3
(a) Na < Mg > Al < Si (b) Na > Mg > Al > Si
(c) E1 = E2 = E3 (d) E1 > E2 < E3
(c) Na < Mg < Al < Si (d) Na > Mg > Al < Si
3. Which of the following is a favourable factor for cation
formation? 9. The electronic configuration of gadolinium (Atomic number
64) is:
(a) High electronegativity (b) High electron affinity
(a) [Xe] 4f3 5d5 6s2 (b) [Xe] 4f7 5d2 6s1
(c) Low ionisation potential (d) Smaller atomic size
7 1
(c) [Xe] 4f 5d 6s 2 (d) [Xe] 4f8 5d6 6s2
4. Generally, the first ionisation energy increases along a
period. But there are some exceptions. The one which is not 10. The statement that is not correct for periodic classification of
an exception is elements is:
(a) Na and Mg (b) Be and B (a) The properties of elements are periodic function of their
(c) N and O (d) Mg and Al. atomic numbers
(b) Non-metallic elements are less in number than metallic
5. Consider the isoelectronic species, Na+, Mg2+, F– and O2–.
elements
The correct order of increasing length of their radii is:
– (c) For transition elements, the 3d-orbitals are filled with
(a) F < O2– < Mg2+ < Na+ electrons after 3p-orbital and before 4s-orbitals

(b) Mg2+ < Na+ < F < O2– (d) The first ionisation enthalpies of elements generally

(c) O2– < F < Na+ < Mg2+ increase with increase in atomic number as we go along a

(d) O2– < F < Mg2+ < Na+ period
Classification of Elements ... 59
11. Among halogens, the correct order of amount of energy 19. The elements with atomic numbers 35, 53 and 85 are all:
released in electron gain (electron gain enthalpy) is: (a) Noble gases (b) Halogens
(a) F > Cl > Br > I (b) F < Cl < Br < I (c) Heavy metal (d) Light metals
(c) F < Cl > Br > I (d) F < Cl < Br < I 20. Electronic configuration of four elements A, B, C, and D are
12. The period number in the long form of the periodic table is given below:
equal to: (a) 1s2 2s2 2p6 (b) 1s2 2s2 2p4
(a) Magnetic quantum number of any element of the period 2 2 6
(c) 1s 2s 2p 3s 1 (d) 1s2 2s2 2p5
(b) Atomic number of any element of the period Arrange them in increasing order of electron gain enthalpies.
(c) Maximum principal quantum number of any element of (a) A < C < B < D (b) A < B < C < D
the period (c) D > B < C < A (d) A > B < C > D
(d) Maximum azimuthal quantum number of any element of 21. Which one of the following represents a d-block element?
the period (a) [Rn] 6d10 7s2 7p2
13. The elements in which electrons are progressively filled in (b) [Xe] 4f1 5d1 6s2
4f-orbital are called:
(c) [Xe] 4f14 5d1 6s2
(a) Actinoids (b) Transition elements
(d) [Xe] 5d1 6s2
(c) Lanthanoids (d) Halogens
22. The electronic configuration having maximum difference in
14. Which of the following is the correct order of size of the first and second ionization energies is
given species:
(a) 1s2 2s2 2p6 3s2 (b) 1s2 2s2 2p6 3s2 3p1
(a) I > I– > I+ (b) I+ > I– > I (c) 1s2 2s2 2p6 3s2 3p2 (d) 1s2 2s2 2p6 3s1
+
(c) I > I > I (d) I– > I > I+
23. The successive ionization energies for element X is given
15. The element with atomic number 57 belongs to: below
(a) s - block (b) p - block IE1: 250 kJ mol–1
(c) d - block (d) f - block IE2: 820 kJ mol–1
16. The last element of the p-block in 6th period is represented IE3: 1100 kJ mol–1
by the outermost electronic configuration: IE4: 1400 kJ mol–1
(a) 7s2 7p6 (b) 5f14 6d10 7s2 7p0 Find out the number of valence electrons for the element X.
(c) 4f14 5d10 6s2 6p6 (d) 4f14 5d10 6s2 6p4 (a) 3 (b) 4 (c) 2 (d) 1
17. Which of the elements whose atomic numbers are given 24. What is the value of electron gain enthalpy of Na+ if IE1 of
below, cannot be accommodated in the present set up of the Na = 5.1 eV?
long form of the periodic table? (a) +2.55 eV (b) +10.2 eV
(a) 107 (b) 118 (c) 126 (d) 102 (c) –5.1 eV (d) –10.2 eV
18. The electronic configuration of the element which is just 25. An element X with Z = 112 has been recently discovered. To
above the element with atomic number 43 in the same group which group and period will it belong, respectively-
is: (a) 12, 7 (b) 10, 6
(a) 1s2 2s2 2p6 3s2 3p6 3d5 4s2 (c) 8, 4 (d) 6, 6
(b) 1s2 2s2 2p6 3s2 3p6 3d5 4s3 4p6 26. Which of the following has lowest Ist ionisation energy?
(c) 1s2 2s2 2p6 3s2 3p6 3d6 4s2 (a) Li (b) C
(d) 1s2 2s2 2p6 3s2 3p6 3d7 4s2 (c) O (d) F

Multiconcept MCQs
1. Ionisation energies of element A are given below in kJ/mol 2. In 4th period of periodic table, how many elements have one
IE1 = 120 , IE2 = 430 , IE3 = 540 or more 4d electrons?
If A reacts with different elements, which compound of A is (a) 2 (b) 0 (c) 13 (d) 6
not possible? 3. Few reactions are given below:
1. O( g ) + e  → O(−g ) , ∆1H 2. F( g ) + e − → F(−g ) , ∆ 2 H

(a) AF (b) A2O
(c) A3N (d) A3N2 3. Cl( g ) + e 
→ Cl(−g ) , ∆ 3 H 4. O(−g ) + e − 

→ O(2g−) , ∆ 4 H

60 Dropper NEET

Which of the following statement(s) is/are correct with (c) Amongst isoelectronic species, smaller the positive
respect to the reactions mentioned above? charge on the cation, smaller is the ionic radius
(a) ∆3H is more –ve than ∆1H and ∆2H (d) Amongst isoelectronic species, greater the negative
(b) ∆1H is less negative than ∆2H charge on the anion, larger is the ionic radius
(c) ∆1H , ∆2H and ∆3H are –ve whereas ∆4H is +ve 11. The diagram below is a part of the skeleton of the periodic
(d) All are correct tables in which elements are indicated by letter which are not
their usual symbols.
4. Which of the following statement is not correct?
(a) Ionisation enthalpy for a neutral atom is always positive
(b) 2nd I.E. of monovalent cation is equal to the first I.E. of
bivalent cation.
(c) Ionisation enthalpy of an ion is always positive
The correct option is :
(d) Both (a) and (b)
(a) 'D' has maximum electron affinity
5. Which of the following is/are incorrect?
(b) 'B' exist in nature in a liquid state
(a) I.P. of O(g) is less than I.P of O( g )

(b) I.P. of Ne(g) is greater than I.P. of Ne+(g) (c) 'A' is a 5th period element in a periodic table
(c) I.P. of N(g) is greater than I.P. of N ( g )
+ (d) (b) & (c)
(d) All are incorrect 12. From the following given electronic configuration, identify
6. Which of the following reactions will involve release of energy? the correct order of electron affinity:
(I) [He] 2s2 2p5 (II) [He] 2s2 2p3
→ S(2g−)
(a) S( g ) 

(III) [Ne] 3s2 3p5 (IV) [Ne] 3s2 3p3


(b) N ( g ) 
→N −
(g)
(a) I > II > III > IV (b) III > I > IV > II
(c) O + e– → O–
(g)   (g)      (s) (c) I < II < III < IV (d) II > III > IV > I
2+
→ Al3( g+)
(d) Al( g )  13. Few elements are matched with their successive ionisation
energies. Identify the elements.
7. Out of N, O, Ne, Na and Na+, which of these species will have
the maximum and minimum ionization energy respectively: Element IE1 (kJ/mol) IE2 (kJ/mol)
(a) Na+ , O (b) Na+ , Na X 2372 5251
(c) N , Ne (d) Ne , N Y 520 7297
8. Which of the following is/are correct with respect to Z 900 1758
reactions given below?
X Y Z
→ M (+g ) + e − ; ∆H = 100 eV
M ( g )  (a) A noble gas Alkali metal Alkaline earth metal
→ M (2+g ) + e − ; ∆H = 250 eV
M + ( g )  (b) Alkali metal A noble gas Alkaline earth metal
(c) Alkaline earth Alkali metal A noble gas
(a) ∆H1 of M(g) is 100 eV (b) ∆H1 of M (+g )is 150 eV
metal
(c) ∆H2 of M(g) is 250 eV (d) All are correct (d) Alkali metal Alkaline earth A noble gas
metal
9. Which of the following statement is incorrect?
(a) The first ionization potential of nitrogen is greater than 14. In the given graph, a periodic property (R) is plotted against
that of oxygen atomic numbers (Z) of the elements. Which property is
(b) The electron affinity of fluorine is greater than that of shown in the graph and how is it correlated with reactivity of
chlorine the elements?
(c) The first ionization potential of Mg is greater than
aluminium
(d) The electronegativity of fluorine is greater than that of
chlorine
10. Identify the wrong statement in the following.
(a) Atomic radius of the elements increases as one moves
down the first group of the periodic table
(b) Atomic radius of the elements decreases as one moves
across from left to right in the 2nd period of the periodic
table
Classification of Elements ... 61
(a) Ionisation enthalpy in a group decrease reactivity (c) The first I.E. of elements with atomic number 1 to 5
decreases from a → e. (d) Successive I.E. for transition elements with four electrons
(b) Ionisation enthalpy in a group decreases reactivity in d-subshell.
increases from a → e.
17. In a periodic table, the basic character of oxides
(c) Atomic radius in a group decrease reactivity decreases
from a→ e. (a) Increases from left to right and decreases from top to
(d) Metallic character in a group decrease reactivity increases bottom
from a → e. (b) Decreases from right to left and increases from top to
bottom
15. Match the entries of List I with appropriate entries of List II
(c) Decreases from left to right and increases from top to
and select the correct answer using the codes given below the
lists: bottom
(d) Decreases from left to right and increases from bottom to
List I List II
top
P. Rutherfordium (At. No. = 1. Period number = 7
18. Identify the incorrect statement.
104)
(a) The first ionisation potential of Al is less than the first
Q. Roentgenium (At. No. = 2. Group number = 4
ionisation potential of Mg.
111)
(b) The second ionisation potential of Mg is greater than the
R. Thorium (At. No. = 90) 3. d-block elements
second ionisation potential of Na.
S. Neptunium (At. No. = 93) 4. f-block elements
(c) The first ionisation potential of Na is less than the first
P Q R S ionisation potential of Mg.
(a) 3 2,4 2,3 1,2,3 (d) The third ionisation potential of Mg is greater than that ofAl.
(b) 1,2,3 1,3 1,4 1,4 19. Elements X, Y and Z have atomic numbers 19, 37 and 55
(c) 1,2 2,1 4,3 3,1 respectively. Which of the following statements is true about
(d) 3,1 1,3 2,4 4,1,2 them?
16. Five ionization energy values in kJ/mol are listed below: (a) Their ionization potential would increase with increasing
E1 = 870, E2 = 830, E3 = 1010, E4 = 1290, E5 = 376. These atomic number.
are (b) Y would have an ionization potential between those of X
(a) Successive ionization energies for the element with and Z.
atomic number 5
(c) Z would have the highest ionization potential.
(b) The first I.E. of successive elements in group 15, 16, 17,
18 and 1 respectively (d) Y would have the highest ionization potential.

NEET Past 10 Years Questions


1. From the following pairs of ions which one is not an iso- 3. For the second period elements the correct increasing order
electronic pair? (2021) of first ionisation enthalpy is: (2019)
(a) Na+, Mg2+ (a) Li < Be < B < C < N < C < F < Ne
(b) Mn2+, Fe3+
(b) Li < B < Be < C < O < N < F < Ne
(c) Fe2+, Mn2+
(c) Li < B < Be < C < N < O < F < Ne
(d) O2–, F–
2. Identify the incorrect match (2020)
(d) Li < Be < B < C < O < N < F < Ne

Name IUPAC Official Name 4. The element Z = 114 has been discovered recently. It will
(A) Unnilunium (i) Mendelevium belong to which of the following family group and electronic
configuration? (2017-Delhi)
(B) Unniltrium (ii) Lawrencium
(C) Unnilhexium (iii) Seaborgium (a) Nitrogen family, [Rn] 5f146d107s27p6

(D) Unununnium (iv) Darmstadtium (b) Halogen family, [Rn] 5f146d107s27p5


(a) (B), (ii) (b) (C), (iii) (c) Carbon family, [Rn] 5f146d107s27p2
(c) (D), (iv) (d) (A), (i) (d) Oxygen family, [Rn] 5f146d107s27p4
62 Dropper NEET
5. In which of the following options the order of arrangement does 8. The species Ar, K+ and Ca2+ contain the same number of
not agree with the variation of property indicated against it? electrons. In which order do their radii increase? (2015)
 (2016 - I) 2+ +
(a) Ca < Ar < K
(a) Li < Na < K < Rb (increasing metallic radius)
(b) Ca2+ < K+ < Ar
(b) Al3+ < Mg2+ < Na+ < F─ (increasing ionic size)
(c) K+ < Ar < Ca2+
(c) B < C < N < O (increasing first ionization enthalpy)
(d) Ar < K+ < Ca2+
(d) I < Br < Cl < F (increasing electron gain enthalpy)
6. The formation of the oxide ion, O2– (g) from oxygen atom 9. Be2+ is isoelectronic with which of the following ions?(2014)
requires first and exothermic and then an endothermic step as (a) Li+ (b) Na+
shown below: (c) Mg2+ (d) H+
O (g) + e– → O– (g) ; ΔfHo = –141 kJ mol–1
10. Which of the following orders of ionic radii is correctly
O– (g) + e– → O2– (g) ; ΔfHo = +780 kJ mol–1 represented? (2014)
Thus, process of formation of O2– in gas phase is unfavourable + –
(a) Na > F > O 2–
even though O2– is isoelectronic with neon. It is due to the
(b) O2– > F– > Na+
fact that, (2015 Re)

(a) O ion has comparatively smaller size than oxygen atom (c) Al3+ > Mg2+ > N3–
(b) Oxygen is more electronegative (d) H– > H+ > H
(c) Addition of electron in oxygen results in larger size of 11. Identify the wrong statement in the following: (2012 Pre)
the ion (a) Atomic radius of the elements decreases as one moves
(d) Electron repulsion outweighs the stability gained by across from left to right in the 2nd period of the periodic
achieving noble gas configuration table
7. The number of d-electrons in Fe2+ (Z = 26) is not equal to the (b) Amongst isoelectronic species, smaller the positive charge
number of electrons in which one of the following? (2015) on the carbon, smaller is the ionic radius
(a) p-electrons in Cl (Z = 17) (c) Amongst isoelectronic species, greater the negative
(b) d-electrons in Fe (Z = 26) charge on the anion, larger is the ionic radius
(c) p-electrons in Ne (Z = 10) (d) Atomic radius of the elements increases as one moves
(d) s-electrons in Mg (Z = 12) down the first group of the periodic table.

Classification of Elements ... 63


ANSWER KEY

Topicwise Questions

1. (a) 2. (a) 3. (b) 4. (a) 5. (b) 6. (a) 7. (a) 8. (b) 9. (c) 10. (a)
11. (d) 12. (d) 13. (d) 14. (d) 15. (b) 16. (d) 17. (c) 18. (b) 19. (d) 20. (d)
21. (c) 22. (c) 23. (c) 24. (c) 25. (b) 26. (c) 27. (d) 28. (d) 29. (c) 30. (b)
31. (c) 32. (a) 33. (b) 34. (d) 35. (d) 36. (c) 37. (d) 38. (a) 39. (b) 40. (c)
41. (d) 42. (b) 43. (d) 44. (c) 45. (d) 46. (a) 47. (b) 48. (a) 49. (d) 50. (a)
51. (d)

Learning Plus

1. (b) 2. (a) 3. (c) 4. (a) 5. (b) 6. (d) 7. (a) 8. (a) 9. (c) 10. (c)
11. (c) 12. (c) 13. (c) 14. (d) 15. (c) 16. (c) 17. (c) 18. (a) 19. (b) 20. (a)
21. (d) 22. (d) 23. (d) 24. (c) 25. (a) 26. (a)

Multiconcept MCQs

1. (a) 2. (b) 3. (d) 4. (d) 5. (d) 6. (c) 7. (b) 8. (d) 9. (b) 10. (c)
11. (a) 12. (b) 13. (a) 14. (b) 15. (b) 16. (b) 17. (c) 18. (b) 19. (b)

NEET Past 10 Years Questions

1. (c) 2. (c) 3. (b) 4. (c) 5. (c), (d) 6. (d) 7. (a) 8. (b) 9. (a) 10. (b)
11. (b)

64 Dropper NEET
4 Chemical Bonding and
Molecular Structure

Past Years NEET Trend

5
No. of MCQs

0
2021 2020 2019 2018 2017 2016 2015 2014 2013 2012

Investigation Report
TARGET EXAM PREDICTED NO. OF MCQs CRITICAL CONCEPTS
• Molecular orbital theory
NEET 4-5 • Dipole moment, VSEPR Theory

Perfect Practice Plan


Topicwise Questions Learning Plus Multiconcept MCQs NEET Past 10 Years Total MCQs
Questions
193 36 24 41 294
INTRODUCTION 1. The incomplete octet of the central atom: In some compounds,
An attractive force that acts between two or more particles the number of electrons surrounding the central atom is less than
eight. This is especially the case with elements having less than
(atoms, ions or molecules) to hold them together, is known as a
four valence electrons. Examples are LiCl, BeH2 and BCl3.
chemical bond.
Cl
It is a union of two or more atoms to aquire stable inert gas Li:Cl H:Be:H Cl:B:Cl

:
configuration ns2np6. Li, Be and B have 1, 2 and 3 valence electrons only. Some other
In a Chemical Bond both attractive and repulsive forces exist in such compounds are AlCl3 and BF3.
equilibrium.
2. Odd-electron molecules: In molecules with an odd number
Atoms are less stable and more energetic hence they form of electrons like nitric oxide. NO and nitrogen dioxide, NO2, the
molecules by loosing some energy by participating in Chemical octet rule is not satisfied for all the atoms
Bond. .+
N. =O O = N – O–:

:
: :

: :

: :
H + H → H2 + 434.72kJ
Cl + Cl → Cl2 + 239.1kJ 3. The expanded octet/ super octet / hypervalent compound:
In some compounds, the number of electrons surrounding the
Formation of bond is accompanied by decrease in potential energy.
central atom is more than eight due to availability of 3d orbitals
Therefore, Chemical Bond formation is always Exothermic.
(incomplete octet of the central atom).
To explain nature of chemical bond different theories are Some of the examples of such compounds are: PF5 SF6, H2SO4
given, at first KOSSEL & LEWIS proposed the theory. and a number of coordination compounds.
Kossel & Lewis approch chemical bonding: In early days,
the ability of various elements to combine with one another was
expressed in terms of their valency. The concept of valency
was not based on any logical understanding. W. Kossel and G.
N. Lewis were the first to provide some logical explanation of
valency which is based on the inertness of noble gases.
This view later on, came to be known as Octet rule.

OCTET RULE FORMAL CHARGE


(Tendency to acquire noble gas configuration) Formal charge is a factor based on a pure covalent bond formed
It has been observed that atoms of noble gases have little or no by the sharing of electron pairs equally by neighbouring atoms.
tendency to combine with each other or with atoms of other Formal charge may be regarded as the charge that an atom in a
elements. molecule would have if all the atoms had the same electronegativity.
It may or may not approximate the real ionic charge. In case of
It means that these atoms must have a stable electronic
a polyatomic ions, the net charge is possessed by the real ion as
configuration.
a whole and not by a particular atom. It is, however, feasible to
These elements (noble gases) have 8 electrons (ns2 np6) except assign a formal charge on an atom in a polyatomic molecule or
helium which has 2 electrons (1s2) in their outer most shell. ion.
The formal charge of an atom in a polyatomic molecule (or) ion
Element Outer most shell Configuration
may be defined as the difference between the number of valence
Ne 2s22p6 electrons of that atom in an isolated or free state and the number
electrons assigned to that atom in the lewis structure.
Ar 3s2sp6
Qf = [NA – NM] = [NA – NLP – 1/2NBP]
Kr 4s24p6 Qf = Formal charge
Xe 5s25p6 NA = number of electrons in the valence shell in the free atom
NM =  number of electrons belonging to the atom in the
Rn 6s26p6 molecule
NLP = number of electrons in unshared pairs, i.e. number of
It is therefore concluded that ns2np6 configuration in the outer electrons in lone pairs
energy level constitutes a structure of maximum stability or
NBP = number of electrons in bond pairs, respectively.
minimum energy.
Example: The Lewis dot formula of PH3 is
“Tendency of atoms to have eight electrons in their outermost shell
is known as Lewis octet rule”.To achieve inert gas configuration,
atoms lose, gain or share electrons.

Limitations of the Octet Rule Formal Charge of P


It applies mainly to the second period elements of the periodic Qf = [NA – NM] = [NA – NLP – 1/2 NBP]
table. There are three types of exceptions to the octet rule: {5-2-1/2(6)} = (5-5) =0
66 Dropper NEET
Formal Charge of H Electronegativity of Changes occuring Nature of the
Q1 = [NA – NM] = [NA – NLP – 1/2 NBP] Elements in the bond formed
valence electrons
= (1-0-2/2)=0
Low High Transfer of e– Ionic bond
● Formal charges on oxygen atoms of ozone
High High Sharing of e– Covalent
bond
3 Low Low Sea of e– molecular Metallic bond
● Formal charge of oxygen (1) = +1 orbital bond
● Formal charge of side oxygen atom(2) = 0
Bond Bond Energy per mole
● Formal charge of side oxygen atom (3) = –1
Ionic bond –––––––––– 700 – 4000 kJ
The formal charge on an atom may or may not be same. It changes
Covalent and coordinate bond –––––––––– 200 – 500 kJ
with the structural environment of the atom in the molecule. In
resonating structures electronic environment changes. Hence the Hydrogen bonding –––––––––– 10 – 40 kJ
formal charge also may change. Van der–waal’s –––––––––– 2 – 10 kJ
Example: Resonating structures of N2O
ELECTROVALENT OR IONIC BOND
Example: N = N = O ↔ N≡N→O
1 2 3 4 5 6
The strong electrostatic forces of attraction between two oppositely
charged ions which are formed due to transfer of electrons from
1
Qf 1 =5 − 4 − ( 4 ) =−1 one atom to another is called Ionic Bond (or) Electrovalent Bond.
2
Electrovalent bond is not possible between similar atoms. This
1 type of bonding requires two atoms of different nature. One atom
Qf 2 =5 − 0 − ( 8 ) =+1
2 should have the tendency to loose electrons i.e. electropositive
in nature and the other atom should have the tendency to accept
1
Qf 3 = 6 − 4 − ( 4) = 0 electrons i.e. electronegative in nature.
2
Formation of Ionic bond is a redox process, because one atom
1 undergoes oxidation and other one undergoes reduction.
Qf 4 = 5 − 2 − (6) = 0
2
Mg → Mg+2+ 2e–
1 2F + 2e– → 2F–
Qf 5 =5 − 0 − (8) =+1
2
Mg+2 + 2F– → MgF2 or Mg+2 (F–)2
1
Qf 6 =6 − 6 − ( 2 ) =−1 Example: IA and VII A group elements form strong ionic
2 compound.
The formal charges do not represent real ionic charges. Na + Cl → Na+ + Cl–
Formal charges represent a tendency to build up positive (or) 2, 8, 1 2, 8, 7 2, 8 2, 8, 8
negative charges.
(Ne) (Ar)
The lowest energy structure is the one with the smallest formal
charges on the atoms. (configuration) (configuration)
Force of attraction is equal in all direction so ionic bond is non–
 KEY NOTE directional.
The most stable structure is the one which has the smallest A definite three dimensional structure are called crystal lattice.
formal charge on the atoms or zero formal charge on the atom.
Energy released during the formation of one mole crystal lattice
is called lattice energy.
CLASSIFICATION OF CHEMICAL BONDS
e.g. (6.023 × 1023)Na+(g) + (6.023 × 1023) Cl–(g) →
On the basis of electronic valency theory and structure, chemical
bonds can be classified as follows: (6.23 × 1023) NaCl(s) + 788 KJ mol–1 (Lattice energy).
Ionic compounds do not have a molecular formula, they only have
empirical formula.

 KEY NOTE
Š More the distance between two elements in the periodic table
more will be the ionic character of the bond.
Š Total number of electrons lost or gained is called
electrovalency.

Chemical Bonding and Molecular Structure 67


Factors Favouring Formation of Ionic Bonds In size and charge, charge will dominate
1. Ionisation energy (I.E.): Amount of energy required to remove Na2O > NaF
an electron from the outermost orbit of an isolated gaseous atom NaCl < Na2S
to form positive ion or cation is called ionzation energy [energy is (d) Al2O3 Na2O MgO
absorbed so it is an endothermic process] Al2O3 > MgO > Na2O
M + I.E. → M+ + e–
Less Ionisation energy ⇒ Greater tendency to form  KEY NOTE
cation. Born - Haber cycle is used to calculate lattice energy.

Example : Characteristics of Ionic Compounds


(i) Physical state:
● Electrovalent compounds are generally crystalline, hard &
2. Electron affinity: Amount of energy released when an electron
brittle in nature.
is added to an isolated gaseous atom to form negative ion or anion
● These compounds are generally made from ions which are
is called electron affinity [energy is released so it is an exothermic
arranged in a regular way as a lattice structure.
process]
● Thus electrovalent compounds exist as three dimensional
         X + e– → X– + EA
solid aggregates.
High electron affinity ⇒ Greater tendency to form anions ● Normally each ion is surrounded by a number of oppositely
charged ions and this number is called co-ordination number.

LATTICE ENTHALPY
The Lattice Enthalpy of an ionic solid is defined as the energy
required to completely separate one mole of a solid ionic compound
into gaseous constituent ions or energy released during the formation
(ii) Boiling point and melting point: High boiling point and
of one mole crystal from its gaseous constituent ions. For example, melting points are due to strong electrostatic force of attraction.
the lattice enthalpy of NaCI is 788 kJ mol-1. This means that 788 kJ of
(iii) Electrical conductivity: It depends on ionic mobility. In
energy is required to separate one mole of solid NaCI into one mole
solid state there are no free ions so they are bad conductors of
of Na+1 (g) and one mole of Cl–(g) to an infinite distance. electricity.
The solid crystal being 3D; it is not possible to calculate lattice In fused state or aqueous solution free ions are present so they are
enthalpy directly from the interaction of forces of attraction and good conductors of electricity.
repulsion only.
Solid state < Fused state < Aqueous solution (Conductivity order)
Factors associated with the crystal geometry have to be included. (iv) Ionic reactions: Ionic compounds show ionic reactions &
Na+(g) + Cl–(g) → NaCl(s) covalent compounds show molecular reactions.
ΔH = ΔHlattice ⇒ –ve (always) Ionic reactions are fast reactions.
Mg2+(g) + 2Cl– (g) → MgCl2 (s) Example: When NaCl is added to AgNO3 solution, white ppt of
AgCl is formed at once.
Factors affecting L.E. Ag + NO3− + Na + Cl− 
→ Na + NO3− + AgCl ↓
1 white ppt
(i) Lattice energy (L.E.) ∝ r = r+ + r– = interionic distance
r (v) Solubility: Ionic compounds are soluble in polar solvent like
(ii) L.E. ∝ Z+, Z– H2O, HF etc.
Z+ ⇒ charge on cation in terms of electronic charge To explain solubility of ionic compound consider an example of
Z– ⇒ charge on anion in terms of electronic charge NaCl in water.
(iii) Size of cation ↑ L.E. ↑ H2O is polar solvent it can be represented as

Lattice energy of Factor
(a) NaCl > KCl (size) ● The Na+ ions gets associated with partially negatively
charged ‘O’ of water.
(b) NaCl < MgO (size, charge)
● And Cl– ions get associated with partially positively charged
(c) NaCl < MgCl2 (size) ‘H’ of water.
68 Dropper NEET
● Mobility of the ion: more is the hydration smaller will be the
mobility of the ions.

+ H 2O

Li+(aq) < Na+(aq) < K+(aq) < Rb+(aq) < Cs+(aq).
● Electrical conductance: It is related to mobility so follows the
same order.

– OH 2

COVALENT CHARACTER IN IONIC COMPOUNDS
(FAJAN’S RULE)
No Bond is 100% ionic in nature. It has some percentage of
Thus charge on Na+
and Cl–
decreases and electrostatic force also covalent character which is explained on the basis of Fajan’s
rule.
decreases which leads to free ion
In a binary compound AB, if the electronegativity difference
● Here, attraction force between H2O and Na+ or Cl– (Hydration between the elements A and B is equal to 1.7, the compound AB
energy) > Electrostatic force between ions in lattice (Lattice is 50% ionic.
energy)
Greater than 1.7, the compound AB is an ionic compound less than
● Energy released due to interaction between polar solvent 1.7, the compound AB is a covalent compound.(Exceptionally HF is
molecule and ions of solute is called solvation energy. If covalent compound even though electronegativity diffrence is 1.7)
water used as solvent it is called hydration energy.
● Condition of the solubility of ionic compound in water is Polarising Power
(Hydration energy > Lattice energy) When anion and cation approach each other, the valence shell of
anion is pulled towards cation nucleus and thus shape of anion is
Solvation or Hydration deformed. This phenomenon of deformation of anion by a cation
is known as polarisation and the ability of cation to polarize a
nearby anion is called as polarizing power of cation.

Polarisability: The tendency of anion to get distorted or polarised


by the cation is called its polarisability
Whenever any compound generally ionic or polar covalent is Polarising relates to cation:
dissolved in a polar solvent or in water then, different ions of 1
Polarising power ∝ charge on cation ∝
the compound will get separated and will get surrounded by size
polar solvent molecules. This process is known as solvation or Polarisability relates to anion
hydration. Energy released in this process is known as solvation Polarisability ∝ charge ∝ Size
energy or hydration energy Polarising power increases covalent character increases
The ionic compound will be soluble only if solvation energy Ex: In case of NaCl, MgCl2, AlCl3 the polarisation increases
(H.E.) is more than the lattice energy Na+ < Mg2+ < Al3+
Factors affecting solvation energy or hydration energy In case of AlF3 < AlCl3 < AlBr3 < AlI3
F– < Cl– < Br– < I–
1 1 
S.E ∝ Z+ Z– ∝  + 
Covalent character increases with increase in size of halide ion.
r+ r− Fajan pointed out that greater is the polarization of anion in a
 1  molecule, more is covalent character in it.
∝ 1 −  (nature of solvent) where ∈r is dielectric constant.
∈ More distortion of anion, more will be polarisation then covalent
 r 
character increases.
Greater the polarity, greater will be ∈r
Fajan gives some rules which govern the covalent character in the
∈r ↑ ⇒ 1/∈r ↓, 1–(1/ ∈r) ↑ ⇒ S.E. ↑ ionic compounds, which are as follows:
H2O CH3OH C2H5OH C6H6 (i) Size of cation : Smaller is the cation more is its polarizing
power and thus more will be the polarisation of anion.
∈r 81 34 27 3
Hence more will be covalent character in compound.
Applications of hydration energy Size of cation ∝ 1 / polarisation.
● Size of the hydrated ions: Greater the hydration of the ion e.g. BeCl2 MgCl2 CaCl2 SrCl2 BaCl2
greater will be its hydrated radii. Size of cation increases → Polarisation decreases →
Li+(aq) > Na+(aq) Covalent character decreases
Chemical Bonding and Molecular Structure 69
(ii) Size of anion : Larger is the anion, greater is its polarisability Ionic character increases, melting point increases ; since size of
and, therefore, more will be the polarisation. Thus more will cations increases & size of anion is constant.
be covalent character in compound.
CaF2, CaCl2, CaBr2, CaI2
Size of anion ∝ polarisation
LiF LiCl LiBr LiI Covalent character increases, melting point decreases ; since size
e.g . 

− Size of anion increases of anions increases & size of cation is constant.
− Polarisation increases
− Covalent character increases
COVALENT BOND
(iii) Charge on cation : Higher is the oxidation state of cation, A covalent bond may be defined as the bond formed by mutual
more will be the deformation of anion and thus, more will sharing of electrons between the participating atoms (which are
be covalent character in compound. short of electrons) of comparable electronegativity.
Charge on cation α polarisation.
NaCl MgCl2 AlCl3

Na + Mg 2+ Al 3+
e.g . 

− Charge of cation increases
− Polarisation increases
− Covalent character increases

(iv) Charge on anion : Higher is the charge on anion more will Covalency
be the polarisation of anion and thus more will be covalent
character in the compound. (i) It is defined as the number of covalent bonds formed by the
atom of the element with other atoms.
Charge on anion ∝ polarisation.
(ii) The usual covalency of an element except hydrogen is equal
AlF3 Al2O3 AlN to 8 minus the number of group to which the element
F − O2− N 3− belongs.
e.g .  →
− Charge on anion increases Ex: In NH3 the valency of Nitrogen is 3

− Polarisation increases In CO2 the valency of carbon is 4
− Covalent character increases
Lewis Structure and Covalent Bond
(v) Pseudo inert gas configuration of cation : Cation having (i) Structures in which valence electrons are represented by
pseudo inert gas configuration has more polarizing power dots are called Lewis structures.
than the cation that has inert gas configuration. Thus NaCl (ii) All atoms in the formulae will have eight electrons in its
having inert gas configuration will be more ionic whereas valence shell. H atom is an exception.
CuCl having pseudo inert gas configuration will be more (iii) Lewis dot formulae shows only the number of valence
covalent in nature. electrons, the number and kinds of bonds, but does not
Cu+ = [Ne] 3s2 3p6 3d10 Na+ = 1s2 2s2 2p6 depict the three dimensional shapes of molecules and
polyatomic ions.
18e– 8e–
(iv) Lewis formulaes are based on the fact that the representative
Pseudo inert gas configuration Inert gas configuration
elements achieve a noble gas configuration in most of their
(poor shielding of d-electrons) (more shielding of s and p electrons) compounds i.e. 8 electrons in their outermost shell (except
for H2, Li+ and Be2+ ions which have only 2 electrons)
Application & Exceptions of Fajan’s Rules
(i) Ag2S is less soluble than Ag2O in H2O because Ag2 S is No. of valence Example Lewis structure
more covalent due to bigger S2– ion. electrons
(ii) Fe(OH)3 is less soluble than Fe(OH)2 in water because Fe3+ 1 Hydrogen/Group IA H./Li.
is smaller than Fe2+ and thus charge is more. Therefore,
Fe(OH)3 is more covalent than Fe(OH)2 . 2. Helium/Group IIA He:/Mg:
(iii) The colour of some compounds can be explained on the 3. Group IIIA B
basis of polarisation of their bigger negative ions.
(iv) Variation of melting point 4. Group IV A C
[melting point of covalent compound< melting point of ionic
: : : :

5. Group V A N
compound]
BeCl2, MgCl2, CaCl2, SrCl2, BaCl2 6. Group VI A :O

70 Dropper NEET
Factors Favouring the Formation of Covalent Bond Molecular solids: They are formed when one atom combines
(1) Small size of cation with another by a covalent bond and then the molecule combines
(2) Large size of anion with another similar molecule with the help of Van der waal’s
(3) Large charge on cation and Anion force of attraction or hydrogen bond.
(4) Generally a covalent bond is formed between 2 atoms of Example : CH4(Solid), dry ice, ice.
similar E.N. values
(iii) Melting and boiling points: With the exception of a few
High ionisation enthalpy: Atoms having high ionisation which have giant three dimensional structures such as diamond,
enthalpies do not have a tendency to lose electrons to form cations.
carborundum (SiC), Silica (SiO2), others have relatively low
Such elements prefer to form covalent bonds. Eg: Cl-Cl
melting and boiling points.
Comparable electronegativity: Atoms with equal or nearly
equal electronegativities tend to share equally a pair of electrons This is due to the presence of weak attractive forces between the
with opposite spins. This leads to a better overlap of their atomic molecules.
orbitals thus resulting in the formation of a covalent bond. (iv) Electrical conductivity: In general, covalent substances are
Note: Sidgwick concept of maximum covalency bad conductors of electricity. Since they do not contain charged
The maximum covalency of an element actually depends on the particles or free electrons.
period of the periodic table to which it belongs. Substances which have polar character like HCl in a solution, can
Period Elementt Max. Covalency conduct electricity.

1 H 2 Graphite can conduct electricity since electrons can pass from one
layer to other.
2 Li to F 4
Some show conductivity due to self ionisation, example:
3 Na to Cl 6
Liq.NH3, H2O, etc.
4 K to Br 6
NH3 + NH3 → NH4+ + NH2–
5 Rb onwards and rest of the 8
6 elements H2O + H2O → H3O+ + OH–

 KEY NOTE (v) Chemical reactions: Covalent substances give molecular


reactions. Reaction rate is usually low because it involves
This rule explains the formation of PCl5 and SF6. This also
two steps (i) breaking of covalent bonds of the reactants (ii)
explains why Nitrogen does not form NF5 or NCl5. Because
Nitrogen belongs to second period and the maximum covalency establishing new bonds. While in ionic reactions there is only
of Nitrogen is 4. regrouping of ions.
(vi) Solubility: Non polar compounds are soluble in non polar
Characteristics of Covalent Compounds solvents. Non polar solvents are CCl4, benzene, CS2 etc.
(i) Physical state: Under normal temperature and pressure, they Polar compounds are soluble in polar solvents (i.e. ΔE.N. is 0.9 to
exist as gases or liquids of low boiling points.
1.8). Polar solvents are H2O, CHCl3, alcohol etc.
This is due to the fact that very weak forces of attraction (Van
(vii) Isomerism: Covalent bonds are rigid and directional. On
der waal’s forces) exist between the molecules due to which
molecules are far from each other. account of this there is a possibility of different arrangement of
atoms in space. Covalent compounds can thus shows isomerism
If their molecular masses are high they exist as soft solids e.g.
sulphur, phosphorus and iodine. (structural and spatial).

(ii) Crystal structures: Covalent solid – In this type of structure MODERN CONCEPT OF COVALENT BOND (VBT)
every atom is bonded to four other atoms by single covalent bonds
resulting in the formation of a giant structure e.g. SiC, AlN and Enthalpy Diagram for H2 Molecule
diamond. These crystals are very hard and possess high melting When two hydrogen atoms are at an infinite distance from each
point. other, there is no interaction between them and therefore, enthalpy
Diamond is sp3 hybridized carbon atom and it forms a tetrahedral of the system is assumed to be zero in this state (stage-A). As
structure. the two atoms start coming closer to each other, the potential
Graphite is sp2 hybridized carbon atoms and it forms hexagonal enthalpy continues to decrease (stage B). Ultimately a stage is
layers which can slide over each other due to weak Van der reached when the enthalpy of the system becomes minimum and
waal’s forces of attraction. Distance between C–C atom is 1.42 hydrogen atoms are said to be bonded together to form a stable H2
Å distance between layers is 3.6 Å. Graphite is more stable than
molecule (stage C).
diamond due to high value of change in enthalpy.
Chemical Bonding and Molecular Structure 71
If both orbitals having opposite sign, called out of phase
overlap (destructive). And if both orbitals are out of phase
Energy (kJ/mol) due to the direction of approach then zero overlap.

r0
0
Distance of separation

Bond
435.8 Energy
Bond Length 74 pm Internuclear distance
The internuclear distance r0 between two hydrogen atoms at this
stage is referred to as bond length. In the case of the hydrogen
molecule, the bond length is 74 pm. It should be noted that two
hydrogen atoms can not be brought at a distance lesser than ro
(i.e., 74 pm) because the potential enthalpy of the system increases Types of Overlapping and Nature of Covalent Bonds
and curve shows an upward trend (dotted lines) and molecule ● Linear overlapping of atomic orbitals results in the formation
becomes unstable.
of sigma bond (σ).
VBT cannot explain paramagnetic behavior of O2. ● A covalent bond formed by the sidewise overlap of atomic
orbitals of atoms already bonded through a ‘σ’ bond and in
Main Points of Valence Bond Theory which the electron cloud is present on either side of the inter
(i) A covalent bond is formed by partial overlapping of two nuclear axis is known as a pi bond (π).
atomic orbitals ● Lateral overlapping of atomic orbitals results in the formation
of pi bond (π).
● ‘π’ bond is formed only after the formation of ‘σ’ bond.
● Any type of orbitals can involve in ‘σ’ bond formation.
● Only ‘p’ or ‘d’ orbitals can involve in ‘π’ bond formation.
● Single bond is equal to one ‘σ’ bond.
● Double bond is a combination of one ‘σ’ bond and one ‘π’ bond.
(ii) More is the extent of overlapping between the two atomic ● Triple bond is a combination of one ‘σ’ bond and two ‘π’ bonds.
orbital, stronger will be bond. Strength of Sigma and pi Bonds
Strength of the bonds follows the order
triple bond > double bond > single bond.
Strength of the bonds follows the order : σp–p > σs–p > σs–s > π
[Principal Quantum no. same, n = 2]
Q s orbital are spherical in nature so they are least diffused  KEY NOTE
hence it will provide less area for overlapping. ‘σ’ bond between ‘1s’ orbitals is exceptionally stronger
(iii) Orbitals which are undergoing overlapping must be such
that Difference in σ and π Bonds
(a) Each orbital should have one electron with opposite
Sigma (σ) bond Pi (π) bond
spin (for formation of covalent bond).
(b) One orbital have pair of electron and the other orbital 1. It results from the end to It result from the sidewise (lat-
have no electron (for formation of co-ordinate bond). end overlapping of two s eral) overlapping of two p or-
orbitals or two p–orbitals bitals.
(iv) If the overlapping is along the molecular axis then bond will or one s and one p-orbital
be sigma (σ) & in the perpendicular direction, it will be pi(π)
bond. 2. Its bonded orbital consists Its bonded orbital consists of
of a single electron cloud two electron clouds one above
symmetrical about inter- and the other below the plane
nuclear axis of symmetry
3. Strong Weak
4. Bond energy 80 kcals 65 kcals
5. More stable Less Stable
(v) If two atomic orbitals having same sign (+ or – ) overlap with
each other, it is called in phase overlapping (constructive). 6. Less reactive More reactive

72 Dropper NEET
Where two or more resonance structures can represent a molecule,
Sigma (σ) bond Pi (π) bond
the VSEPR model is applicable to any such structure.
7. Can independently exists Always exists along with a σ The repulsion between electron pairs on central atom follows the
bond hybridsation order
8. Hybridization depends doesn’t depend on p bond l.p-l.p>l.p-b.p>b.p-b.p.
upon σ bond In VSEPR theory the number of electron pairs in single bond or
9. The groups or atoms can Due to resistance to rotation double bond or triple bond or dative bond is counted as only one
undergo bond rotation around the π bond the groups pair because all the electron pairs in the same bond are oriented in
about single sigma (σ) attached to it are not free to ro- the same direction.
bonds tate. The magnitude of repulsions between bond pair of electrons
depends on the electronegativity difference between the central
10. The σ electrons are referred In π bond the electrons are held atom and other atom.
as localised less firmly and thus can be easi-
Identification of hybridisation and shape of a molecule or ion
ly dissociated or polarised by an
having single central atom.
external charge and hence the π
– Rules:
e are referred as mobile elec-
trons. Step-1: Calculate valence electrons in central atom.
11. Shape of the molecule is π bonding does not affect the Step-2: If it is cation then substract charge from central atom
determined by the σ bonds shape of the molecule Step-3: If it is anion then add charge to the central atom.
present in the molecule Step-4: Bond pairs = No.of atoms attached to central atom.

TRAIN YOUR BRAIN Step-5: Lone pairs =


 No. of electrons given by bonded atoms 
Q. Identify the correct statement : (Valence e– on central atom) –  
 2 
(a) Single N–N bond is stronger than single P–P bond
Step-6: B.P+L.P = Electron Pairs
(b) Single N–N bond is weaker than single P–P bond
2 3 4 5 6 7
(c) N ≡ N is weaker than P ≡ P
↓ ↓ ↓ ↓ ↓ ↓
(d) None of these
sp sp2 sp3 sp3d sp3d2 sp3d3
Ans. (b) Due to small size of nitrogen, the lp–lp repulsion is more
than that in P. Hence statement B is correct
Q. How many sigma and pi bonds are present in  KEY NOTE
tetracyanoethylene? If bonded atoms are H, F then each one contribute one electron
(a) Nine σ and nine π (b) Five π and nine σ for bond formation. If bonded atoms are O, S then each one
(c) Nine σ and seven π (d) Eight σ and eight π contribute two electrons for bond formation.
Ans. (a)
N C C N Shape (molecular geometry) of Some Simple
C C Molecules/Ions
N C C N

VALENCE SHELL ELECTRON PAIR REPULSION No. of No of Formula Molecular Example


(VSEPR) THEORY B.Ps L.Ps shape
Postulates 2 0 AB2 Linear BeCl2, BeF2,
The shape of a molecule depends upon the number of valence C2H2
shell electron pairs (bonded or non-bonded) around the central 3 0 AB3 Trigonal BF3, BCl3
atom. Planar
Pairs of electrons in the valence shell repel one another since their 4 0 AB4 Tetrahedral CH4, NH4+,
electron clouds are negatively charged. CCl4
These pairs of electrons tend to occupy such positions in space 5 0 AB5 Trigonal PCl5
that minimise repulsion and thus maximise distance between bipyramidal
them.
The valence shell is taken as a sphere with the electron pairs 6 0 AB6 Octahedral SF6
localising on the spherical surface at maximum distance from one 2 1 AB2E Bent SO2, O3
another.
3 1 AB3E Trigonal NH3
A multiple bond is treated as if it is a single electron pair and Pyramidal
the two or three electron pairs of a multiple bond are treated as a
single super pair. 2 2 AB2E2 Bent H2O

Chemical Bonding and Molecular Structure 73


4 1 AB4E See Saw SF4 HYBRIDISATION
3 2 AB3E2 T-Shape ClF3 ● The concept of hybridisation was introduced by Pauling
● The intermixing of atomic orbitals of almost same energy and
5 1 AB5E Square BrF5
their redistribution into an equal number of identical orbitals
Pyramidal
is known as hybridisation
4 2 AB4E2 Square XeF4
● The orbitals of one and the same atom only involve in
Planer
hybridisation.
A = Central atom in the compound. ● The no.of hybrid orbitals formed is numerically equal to
B = atom linked to the central atom no.of orbitals participating in hybridisation.
E = Lone pairs of electron ● The hybrid orbitals symmetrically arranged around the
nucleus such that they have maximum stability.
TRAIN YOUR BRAIN ● The order of repulsions will be in order of lp-lp> lp-bp>bp-
bp repulsions.
Q. The interhalogen compound that cannot exist is
● The orbitals involving in the hybridisation have different
(a) IBr3 (b) ICl7 (c) IF4 (d) BrF5 shapes but almost same energy.
Ans. (c) I does not have its valency 4. It has valency 1, 3, 5 & 7 ● The orbitals formed in hybridisation process are called hybrid
Q. How many number of species do not exist :- orbitals.
PCl5 , PH5 , SF6 , SH6 , PBr6–, OF4 ● The hybrid orbitals have same shape and same energy.
● The angle between any two hybrid orbitals in an atom is
(a) 2 (b) 3
generally same.
(c) 4 (d) 5
● Electron filling in hybrid orbitals obeys Hunds rule and
Ans. (c) PH5, SH6 → not exist due to non expandable covalency Pauli’s rule.
OF4 → O-atom does not have vacant d-orbital. ● The hybrid orbitals involve only in σ bond formation. They
PBr6– → steric repulsion never involve in ‘π’ bond formation.
● The concept of hybridisation was introduced to explain the
TRAIN YOUR BRAIN shapes of molecules, bond angles and bond lengths in the
Q. Which of the following pairs of species have identical molecules.
shapes?
● A half filled or completely filled or even vacant orbital can
(a) NO2+ and NO2– (b) PCl5 and BrF5 participate in Hybridization.
(c) XeF4 and ICl4– (d) TeCl4 and XeO4
● The hybrid orbitals are more effective in forming stronger
bonds that leads to the formation of more stable molecules.
Ans. (c) (a)
 KEY NOTE
Same atom undergoes different types of hybridisation under
different situations.
More the directional bond, greater is the bond strength
(b)
sp3 – sp3>sp2–sp2>sp–sp>p–p>s–p>s–s

sp-hybridisation
● This is also known as diagonal hybridisation or linear
hybridisation.
● One s- orbital combines with one p- orbital to give two
identical orbitals called sp - hybrid orbitals.
(c)
● The angle between the two sp-hybrid orbitals in an atom is 180°

1
● sp- hybrid orbital will have s- character or 50% s- character.
2
1
p- character or 50% p- character.
(d) 2
● The hybridisation of central atom in a linear molecule is
See-Saw Tetrahedral generally sp.

74 Dropper NEET
sp2-hybridisation sp3d2- hybridisation
● This is also known as trigonal hybridisation. ● One s-orbital , three p-orbitals and two d- orbitals combines
● One s- orbital combines with two p-orbitals to give three to give six identical orbitals called sp3d2 hybrid orbitals.
identical orbitals called sp2-hybrid orbitals. ● The angle between any two sp3d2 hybrid orbitals is 90° and
180°.
● The angle between any two sp2-hybrid orbitals in an atom is
120°. 1
● sp3d2 hybrid orbital will have th s-character or 16.7% s-
1 1 16
character, p − character & d − character
● sp2 hybrid orbital will have rd s-character or 33.3% 2 3
3
2 3 2
Example for sp d hybridisation is SF6
s-character and rd p- character or 66.7% p-character.
3
● 3dz and 3dx2–y2 orbitals are involved in sp3d2 hybridisation.
● The hybridisation of central atom in a molecule having 2

trigonal planar shape is sp2. Assigning the Type of Hybridisation


sp3- hybridisation (I) Total molecular valency method
Step 1: Total number of valence electron in the molecule is
● This is also known as tetrahedral hybridisation or tetragonal
calculated by adding the individual valency of all constituent atoms.
hybridisation.
Step 2: If total electrons are greater than eight, then divided by
● One s- orbital combines with three p-orbitals to give four
eight and quotient is equal to hybridised orbitals.
identical orbitals called sp3 hybrid orbitals.
e.g:.PCl5
● The angle between any two sp3-hybrid orbitals in an atom is
109°28'. Total valence electrons = 5+(5 × 7)=40
40 / 8=5=no.of hybridised orbitals → sp3d
1 3
● sp3- hybrid orbital will have th s-character or 25% s- character. th If remainder comes, that remainder should be divided by 2, initial
4 4
quotient comes then sum of quotient and final quotient is equal to
p-character or 75% p-character. number of hybridised orbitals, e.g., XeOF2
● The hybridisation of central atom in a molecule having Total valence electrons = 8 + 6 + (2 x 7) = 28
tetrahedral shape is sp3. 28 / 8 Quotient=3 ; Remainder = 4
dsp2 hybridisation Remainder is divided by 2
4 / 2 Quotient = 2
● The orbitals involved in hybridisation are dx2 – y2, s and
two p-orbitals having square planar geometry, bond angle Hence, sum of Quotient = 3+2 = 5 = no. of hybridised orbitals →
between any two dsp2 orbitals is 90°. sp3d
Step 3: If total valence electrons are either eight or less is divided
sp3d-hybridisation by 2,what Quotient appears,should be considered as number of
● One s-orbital, three p-orbitals and one d-orbital combines and hybridised orbitals, e.g., H2O
gives five identical orbitals called sp3d-hybrid orbitals. Total valence electrons = 2 + 6 = 8
● The angle between two sp3d hybrid orbitals in an atom is 90° 8 / 2 = 4 = sp3
and 120°. (II) Total number of Hybridised orbitals(For neutral molecule)
1
● sp3d hybrid orbital will have th s-character or 20% = 1/2(V + M)
5
3 1 V = no. of valence electrons
s-character, th p-character or 60% p-character, th M = no. of monovalent atom
5 5
d-character or 20% d-character.
Total number of Hybridised orbitals(For cationic species)
● The hybridisation of central atom in a molecule having = 1/2(V + M – C)
trigonal bipyramid shape is sp3d.
Total number of Hybridised orbitals(For anionic species)
Example for sp3d hybridisation is PCl5 = 1/2(V + M + A)
● 3dz orbital is involved in sp3d hybridisation.
2 C = +ve charge. A = -ve charge.
● The axial bonds (2.19Å) have been found to be larger than
equatorial bonds (2.04Å) since axial P-Cl bonds experience
Hybridisation in Carbon Compounds
greater repulsion from equatorial P-Cl bonds. ● The carbon atom in carbon compounds is tetravalent (i.e.,
No. of planar atoms in PCl5 is 4. carbon atoms form four covalent bonds).
PCl5 is highly reactive and in the solid state exist as [PCl4]+ ● The sp3 carbon atom involves in four single bonds, C
and [PCl6]– ions.

Chemical Bonding and Molecular Structure 75


● The sp2 carbon atom involves in a double bond and two single The molecular orbitals like atomic orbitals are filled in
accordance with the Aufbau principle obeying the Pauli's
bonds, =C exclusion principle and the Hunds rule.
● The sp carbon atom involves in two double bonds or one The molecular orbitals are named as σ, π, δ etc.
triple bond and one single bond = C = or ≡ C –
According to wave mechanics, the atomic orbitals can be
Types of expressed by wave functions (ψ), which represent the amplitude of
Steric number Geometry
Hybridisation waves. These are solutions of schrodinger wave equation. However
it is difficult to solve the above exactly for many electron systems.
2 sp Linear
To over come this problem scientists have used an approximate
3 sp2 Trigonal planar method known as LCAO (Linear Combination of Atomic Orbitals)
4 sp3 Tetrahedral method.
The atomic orbitals of two hydrogen atoms A and B may be
5 sp3d Trigonal
represented by wave functions ψA and ψB .
bipyramidal
Mathematically, formation of molecular orbitals may be
6 sp3d2 Octahedral
described as
7 sp3d3 Pentagonal ψMO = ψA ± ψB
bipyramidal
σ = ψA + ψB (Bonding molecular orbital)
σ* = ψA – ψB (Antibonding molecular orbital)
TRAIN YOUR BRAIN
Quantitatively, the formation of molecular orbitals can be
Q. In which of the following central atom is unhybridised? understood in terms of the constructive or destructive interference
(a) S(CH3)2 (b) SO2 of the electron waves of the combining atoms.
During the formation of BMO, the two electron waves
(c) SiH4 (d) PCl3
of the bonding atoms reinforce each other due to constructive
Ans. (a) As per Drago’s rule interference.
During the formation of ABMO, the electron waves cancel
MOLECULAR ORBITAL THEORY
each other due to destructive interference.
It was proposed by Hund and Mulliken to explain paramagnetic
As a result,the electron density in a bonding molecular
nature of oxygen molecule.
orbital is located between the nuclei of the bonded atoms but in
The electrons in a molecule are present in various molecular case of antibonding molecular orbital most of the electron density
orbitals as the electrons of atoms are present in the various atomic is located away from the space between nuclei( nodal plane).
orbitals.
Molecular orbitals are formed by linear combination (LCAO)
of atomic orbitals of comparable energies and proper symmetry.
This means that 1s orbital can combine with another 1s orbital
but not with 2s orbital because the energy of 2s orbitals is higher
than that of 1s orbital. This is true only for homonuclear diatomic
molecules.
By convention z-axis is taken as the molecular axis, 2pz
orbital of one atom can combine with 2pz orbital of the other atom
but not with the 2px or 2py orbitals because of their symmetry.
Molecular orbital is polycentric i.e. an electron in molecular
orbital is under the influence of two or more nuclei.
The number of molecular orbitals formed is equal to the
number of combining atomic orbitals. When two atomic orbitals
combine, two molecular orbitals are formed. One is known as
bonding molecular orbital while the other is called anti bonding
orbital.
It may be noted that the bonding molecular orbital is
stabilized to the same extent as the anti bonding molecular orbital
is destablised i.e. energy lost by BMO is equal to energy gained
by ABMO but total energy of two MO equal to sum of energy of
two atomic orbitals.
Order of energies of various orbitals is bonding orbitals < non
bonding orbitals < anti bonding oritals.

76 Dropper NEET
The increasing order of relative energies of M.O having less than Bond order ∝ Bond energy
or equal to 14 electrons. ∝ stability of a molecule
σ1s < σ*1s < σ2s < σ*2s < π2px = π2py < σ2pz < π*2px 1

= π*2py < σ*2pz bond length
for more than 14 electrons 10 − 4
N2
Bond order of= = 3
σ1s < σ*1s < σ2s < σ*2s < σ2pz < [π2px = π2py] <[π*2px 3
9−4
= π*2py] < σ*2pz Bond order of = N 2+ = 2.5
2
Difference between σ and π MO’s 2+ 8 −4
Bond order of N = 2 = 2.0
2
σ-molecular orbital π- molecualr orbital 10 − 5
Bond order of= N 2− = 2.5
1. Formed by the end 1. Formed by the sidewise 2
on overlap along the overlap perpendicular to 10 −6
internuclear axis internuclear axis Bond order of = N 22 − = 2
2
2. Overlapped region is very 2. Overlapped region is As the bond order increases the stability of molecule increase.
large small However, some of the bond orders are identical. In such case, a
3. Rotation about the 3. Rotation about the molecule or ion with more number of electrons in their antibonding
orbitals is less stable. Hence correct order is
internuclear axis is internuclear axis is
symmetrical unsymmetrial N22 – < N22 + < N2– < N2+ < N2.

4. Strong bonds are favoured 4. Weak bonds are The bond orders of 1,2 or 3 correspond to single, double or triple
favoured bond. But bond order may be fractional in some cases.
The magnetic properties of molecules can also be ascertained
 KEY NOTE Bonding in some diatomic molecules and ions
If all the electrons in the molecule or ion are paired it is Hydrogen molecule (H2)
diamagnetic in nature.If it is unpaired that is paramagnetic.
Total number of electrons = 2, filling in molecular orbitals we
Magnetic moment
= µ n ( n + 2 ) B.M have
σ12s < σ1*0s
Stability of Molecules
( Nb − N a )
2−0
If Nb is the number of electrons occupying bonding orbitals and Bond order = = = 1
2 2
Na the number of electrons occupying antibonding orbitals, then
Hence there is a single bond between two hydrogen atoms and
A positive bond order is Nb>Na means a stable molecule. While a due to absence of unpaired electrons it is diamagnetic.
negative (Nb<Na) or zero (Nb=Na) bond order means an unstable
molecule Helium molecule (He2)
Electronic configuration / Bond order of simple diatomic molecules The total number of electrons = 4 and filling in molecular orbitals
The electronic configuration and the bond order in case of simple we have
diatomic molecules can be obtained by filling the molecular σ12s < σ1*2s

orbitals by applying Aufbau principle and Hund's rule etc.
( Nb − N a )
2−2
Bond Order Bond order = = 0
=
2 2
The relative stability of a molecule can be determined on the basis Hence, He2 molecule can not exist.
of bond order. It is defined as the number of covalent bonds in
Nitrogen molecule (N2)
a molecule. It is equal to one half of the difference between the
The total number of electrons =14 and filling in molecular orbitals
number of electrons in the bonding and antibonding molecular
we have
orbitals.
π2 px2 
1 σ12s σ1*2s σ22 s σ*2 σ2
Bond order = [Number of bonding electrons – Number of 2s  2  2 pz
2  π 2 p y
antibonding electrons]
Bond order =
( Nb − N a ) 10 − 4
N − Na = = 3
or, = b 2 2
2 It is diamagnetic.
Chemical Bonding and Molecular Structure 77
Oxygen molecule (O2) Boron Molecule (B2)
Total number of electrons =16 and electronic configuration is No of electrons = 10
The electronic configuration is σ1s2 σ*1s2 σ2s2 σ*2s2 π2p1x = π2py1
π2 px2   π* 2 p1x 
σ12s σ1*2s σ22 s σ*2
2s σ 2
2 pz  2  *
σ*
1  2 pz It has 2 paired electrons. Hence, paramagnetic.
π2 p y   π 2 p y 
Basic Idea of Metallic Bonding
( Nb − N a )
10 − 6
In a metal crystal, all atoms are identical. So, these cannot be
Bond order = = = 2
2 2 linked by electrovalent bonds as all have same E.N..
As shown by electronic configuration the O2 molecule contains two Hence, in metal crystal, the atoms are bonded with each other
unpaired electrons, hence it is paramagnetic in nature. with a special type of bonding known as Metallic bonding.
Since the metals are electropositive, their ionisation evergies are low.
O+2 ion
So they readily lose their valence e–s and convert into positive ions.
Total number of electrons (16 - 1) = 15, These e–s move from place to place through the empty valence orbitals
Electronic configuration and are shared simultaneously among all of them.

π2 px2   π* 2 p1x  Essential Conditions for Metallic Bonding


σ12s σ1*2s σ22 s σ*2
2s σ 2
2 pz  2  *
*
 σ 2 pz
π2 p y   π 2 p y  The metal atoms should have low I.E.
There should be sufficient number of vacant orbitals in the valence
10 − 5 shell.
Bond order = = 2.5
2 Thus, the bonding which holds the metal atoms together on
It is paramagnetic. account of the force of attraction between metal ions & the mobile
e–s is called metallic bonding.
O2– Super oxide ion The strength of metallic bond is directly proportional to the no. of
valence e-s and the charge present on the nucleus. It provides an
Total number of electrons (16 +1) = 17. Electronic fonguration
answer to the fact that alkali metals are soft and have low M.P. &
π2p 2x  π* 2p 2x  B. P.. While Transition metals are hard and have high M.P. & B.P.
2 *2 2
σ1s σ1s σ2s σ*2
2s σ 2
2p z  2  * 1  σ2pz
*

 π2p y   π 2p y 
TRAIN YOUR BRAIN
Bond order =
( N b −=
Na ) 10 − 7
= 1.5 Q. How many number of molecules have bond dissociation
2 2 energy greater than that of I2 ?
It is Paramagnetic. F2, Cl2, Br2, N2, O2
Peroxide ion (O22 – ) (a) 2 (b) 3
(c) 4 (d) 5
Total number of electrons (16 + 2) = 18. The electronic
configuration is Ans. (d) Cl2 > Br2 > F2 > I2
N ≡ N , O = O there for B.D.E is higher than I2
 2

 π2 p x  π* 2 px2  DIPOLE MOMENT
σ12s σ1*2s σ22 s σ*2
2s σ22 pz  2  * 2  σ 2 pz
*

 π2 p y   π 2 p y  When a covalent bond is formed between two similar atoms (For


  ex H2, F2, O2) the shared pair of electrons situated exactly between
two atoms, the bond so formed is called non-polar covalent bond.
10 − 8 If a covalent bond is formed between two dissimilar atoms (ex
Bond order = =1
2 H-F) the shared pair gets displaced more towards Fluorine since
It is diamagnetic. Fluorine is more E.N. than Hydrogen, such bond is called polar
Bond dissociation energy order covalent bond.
Polarity of a polar molecule is expressed in terms of dipole
N2 > N2+ = N2– > N22–
moment (μ).
Bond length order
Dipole moment (μ)
N22– > N2– = N2+ > N2
= charge on the pole × distance between the poles.
Bond dissociation energy order Dipole moment is usually expressed in Debye units (D)
O2+ > O2 > O2– > O22– 1 D = 3.33564 × 10–30 Cm (S.I) where
Bond length order C = coulomb and m = meter
O22– > O2– > O2 > O2+ = 10–18 esu. Cm (CGS)

78 Dropper NEET
Dipole moment is vector quantity and is represented by crossed Net dipole moment of water = 1.85 D
arrow . The cross is on positive end and arrow head is on = 1.85 × 3.336 x 10–30 Cm
negative end. For ex
H–F = 6.17 × 10–30 Cm
In polyatomic molecules the dipole moment not only depends
upon the individual dipole moments of bonds known as bond
dipoles but also on the spatial arrangements of various bonds in
the molecule. In such case, the dipole moment of a molecule is the
vector sum of the bond dipoles of various bonds.

Calculation of Resultant Bond Moment


Let AB & AC are two polar bonds inclined at angle θ, their
In NF3 the bond pairs are moving opp. to lone pair and in NH3

:
dipole moments are μ1 & μ2. The resultant dipole moment may be
the bond pairs moving towards the lone pair.
calculated using µ R = µ12 + µ 22 + 2µ1µ 2 cos θ
Cis and trans isomers can be distinguished by dipole moments,
usually cis isomers have higher dipole moment and hence higher
polarity e.g.

Type of Dipole Mo-


Example Geometry
Molecules ment, μ(D)

HF 1.78 Linear Dipole moment is greatest for ortho isomer, zero for para isomer
which is less than that of ortho or meta isomer.
HCl 1.07 Linear
AB HBr 0.79 Linear
HI 0.38 Linear
H2 0 Linear

H2O 1.85 Bent


AB2 H2S 0.95 Bent
CO2 0 Linear μ = 2.53 μ = 1.48 μ=0
Hybridisation can be determined by dipole moment for e.g.
NH3 1.47 Trigonal-
pyramidal (i) If a molecule AB2 has μ = 0, the σ orbitals used by A (Z <
21) must be sp hybridised e.g. BeF2
NF3 0.23 Trigonal-
AB3
pyramidal F Be F
BF3 0 Trigonal-
(ii) If a molecule AB3 has µ= 0, the σ orbitals used by A (Z <
planar
21) must be sp2 hybridised e.g. BF3
CH3Cl 1.87 Tetrahedral F
AB4 CHCl3 1.04 Tetrahedral F B
CCl4 0 Tetrahedral F
Note:
Applications of Dipole Moment 1. Dipole moment ∝ electronegativity difference
Dipole moment is helpful in predicting the geometry of the HF > HCl > HBr > HI
molecule. 2. Dipole moment ∝ No. of lone pair of electrons.
Dipole moment helps in determining the polarity. HF > H2O > NH3
Dipole moment can distinguish between symmetrical and non 3. For homo atomic molecules and molecules having normal
symmetrical molecules e.g. CO2 has 0 diploe moment as it is shapes such as linear, trigonal, tetrahedral posses zero
symmetrical whereas H2O has a dipole moment of 1.85 D. dipole moment.
Ex: Cl2, N2, O2, CO2
4. Molecules having distorted shapes like angular, pyramidal
shows dipole moment
Ex: H2O, SO2

Chemical Bonding and Molecular Structure 79


Some important Points about Dipole Moment
 KEY NOTE
A polyatomic molecule having polar covalent bonds but zero
Š Resonance energy = Actual bond energy – Energy of most
dipole moment indicates the symmetrical structure of the
stable resonating structure.
molecule. e.g. B–F bonds are polar in BF3 but BF3 has µ = 0 due Š Stability of molecule ∝ resonance energy.
to its symmetrical geometry. Š More is the number of covalent bonds in molecule more will
If molecule have µ = 0, then it should be linear or having be its resonance energy.
Š Resonance energy ∝ number of resonating structures.
symmetrical geometry.
e.g. linear – CO2, CS2, BeCl2 (g) ; symmetrical geometry – BF3, BOND LENGTH
CH4 , PCl5 , SF6 , IF7 , XeF4.
The equilibrium distance between the nuclei of two bonded atoms
If molecule has µ ≠ 0 then it should be angular or having in a molecule is called bond length.
unsymmetrical geometry. Bond length is measured by spectroscopic, x-ray diffraction, and
e.g. SnCl2 , PbCl2, SO2 – angular molecular geometry. electron -diffraction techniques.
NH3, H2O, NF3, SF4, H2S – unsymmetrical molecular geometry. Covalent radius of an atom is equal to half the internuclear
distance between two identical atoms that are held together by
RESONANCE covalent bond.
Representation of molecule in more than one structure and none d A− A
Covalent radius of A =
of them explains all the properties of the molecule singly is called 2
Covalent radius of an atom in heterodiatomic molecule (A-B type)
Resonance.
when the E.N. difference between two atoms is large is calculated
Example: by using pauling empirical equation.
dA–B = rA + rB + C(XA – XB)
Where XA and XB= E.N. of atoms A and B the value of C depends
on the type of atoms involved in covalent bond.
For bonds involving atoms of II nd period, C = 0.09
For bonds between Si, P and S bonded to more E.N atoms not
belonging to first period, C = 0.06.
The bond length between a set of atoms in different molecules is
always same.
O-H bond length in H2O, H2O2, C2H5OH is same i.e. 0.97Å
As the p-character of hybrid orbital changes the bond length
changes.
C-H bond length in C2H6, C2H4 and C2H2 are 1.09Å, 1.08Å and
1.06 Å respectively.
As number of bonds between two atoms increases bond length
decreases.
C – C; C = C; C≡C
1.54 Å; 1.33 Å; 1.20 Å
Bond order = As the size of bonded atom increases bond length increases.
Total No. of bonds formed between two atoms in all structures HF HCl HBr HI
Total No. of resonating structures 92pm 127pm 141pm 160pm

 KEY NOTE
It is effected by resonance and hyperconjugation.

BOND ENTHALPY
2 +1+1 It is the amount of energy required to break one mole of bonds of
Bond order = = 1.33 a particular type between two atoms in a gaseous state.
3
80 Dropper NEET
The unit of bond enthalpy is kJ/mole TRAIN YOUR BRAIN
H2(g) → 2H(g), ΔH°d = 435.8 kJ/mole Q. The ONO angle is maximum in :
(a) HNO3 (b) NO+2
Heterolytic fission requires more bond energy than homolytic (c) HNO2 (d) NO2
fission. 102°

The number of bonds between two atoms increases as bond H 12


1 O
Ans. (b) (a) 141
enthalpy increases. O N 130°

C-C C=C C≡C


114°
O
+
341 610 828 kJ/mole
(b) O = N = O Bond angle is 180º because of sp
115 pm
As the number of lone pairs on a bonded atom increases, the bond hybridisation of nitrogen.
enthalpy decreases. 102°
H
•• •• •• •• (c) 143 (d)
−C − C − > − N − N − > − O − O − O N

11
•• ••

8
111° O
In case of H2O molecule, the enthalpy needed to break two O-H
bonds is not the same due to changed chemical environment. BOND ORDER
Therefore mean or average bond enthalpy is used. ● Bond order indicates number of bonds between two atoms in
–1
a molecule.
H2O(g) → H+(g) + –OH(g), ΔaH1Θ= 502 kJ mol ● Greater the bond order more is the strength of the bond.
–OH –1
→ H+(g) + O2–(g), ΔaH2Θ = 427 kJ mol ● With increase in bond order, bond enthalpy increases and
(g)
bond length decreases.
Average bond enthalpy of O-H bond in water ● Bond order 3 is being one of the highest for a diatomic molecule.

502 + 427 ● Isoelectronic species have same bond order.


= = 464.5 kJ
2 Eg,. N2, CO,NO+
● It is observed that with the increase in bond order, bond
 KEY NOTE energy increases and bond length decreases.
Š The magnitude of bond energy depends upon size of the
atoms forming the bond. HYDROGEN BOND
Š Shorter the bond length higher the bond energy. Bond energy Nitrogen, oxygen and fluorine are the highly electronegative
values are less for homolytic fission when compared to elements. When they are tied to a hydrogen atom to form covalent
heterolytic fission. bond, the electrons of the covalent bond are shifted towards
the more electronegative atom. This partially positive charged
hydrogen atom forms a bond with the other more electronegative
BOND ANGLE atom. This bond is called as hydrogen bond and is weaker than
● Angle between the orbitals containing bonding pair of covalent bond.
electrons around the central atom in a molecule or complex Hydrogen bonding is said to be formed when slightly acidic
ion is called bond angle. hydrogen attached to a strongly electronegative atom such as F,
● Bond angle is determined by spectroscopic methods. N and O is held with weak electrostatic forces by the lone pair of
electrons of the electronegative atom.
● As bond angle increases stability increases.
Hydrogen bond is represented by a dotted line (– – –)
Bond angle is effected by the presence of lone pair of electrons on
For example, in HF molecule, the hydrogen bond exists between
central atom
hydrogen atom of one molecule and fluorine atom of another
Ex: CH4 → 109°28' → [0 L.P] molecule as given below :
NH3 → 107° → [1 L.P]    – – – Hδ+ – Fδ–– – – – Hδ+ – Fδ–– – – – Hδ+ – Fδ–
H2O → 104.5° → [2 L.P]
Hydrogen bond is weaker than covalent bond and stronger than
Bond angle depends on state of hybridisation. van der waal force of attraction.
If the electronegativity of central atom decreases bond angle Hydrogen bond present in the same molecule is known as intra
decreases. molecular hydrogen bond.
In case, the central atom remains same bond angle increases with Due to intramolecular hydrogen bonding, ring formation or
chelation occurs.
the decrease in electronegativity of surrounding atoms.
Substances having intramolecular hydrogen bonds are less water
% s-character increases bond angle increases. soluble and steam volatile and have low boiling points.
Chemical Bonding and Molecular Structure 81
Hydrogen bond formed between different polar molecules is In water molecules: Due to polar nature of H2O there is
known as inter molecular hydrogen bond. association of water molecules giving a liquid state of abnormally
Order of H-bond strength high boiling point.

There is also similar H-bonding in alcohol (R—OH) ammonia


(NH3) and phenol (C6H5OH) molecules.
Types of Hydrogen Bonds The hydrogen bonds in HF link the F atom of one molecule with
the H-atom of another molecule, thus forming a zig-zag chain
(A) Intramolecular H-Bonding: This type of H-bonding
(HF)n in both the solid and also in the liquid.
occurs when polar H and electronegative atom are present
in the same molecule.

Carboxylic acid dimerises in gaseous state due to H-bonding

It has lower boiling point (i.e. more volatile) than its para-
derivative (where association of molecules takes place using
intermolecular H-bonding) because it exists as discrete molecules.
Alcohol is said to be highly soluble in water due to crossed
intermolecular H-bonding (between H2O and R—OH molecules).

However, isomeric ether is less soluble in water due to less polar


nature of ether.

Extra stability of the complex is because of intramolecular
hydrogen bonding in addition to the chelating effect. HCO3– ions exist as dimer in KHCO3
Necessary conditions for the formation of intramolecular
hydrogen-bonding In Na+HCO3–, the HCO3– ions are linked to an infinite chain through
(a) The ring formed as a result of hydrogen bonding should be intermolecular H-bonding.
planar.
(b) 5- or 6- membered ring should be formed.  KEY NOTE
(c) Interacting atoms should be placed in such a way that there Š The strongest H-bonds are formed by F atoms. Deuterium is
is minimum strain during the ring closure. more electropositive than H, therefore it also forms stronger
(B) Intermolecular H-Bonding: It is formed between two bonds.
different molecules of the same or different compounds.

82 Dropper NEET
Topicwise Questions

KOSSEL LEWIS THEORY & LEWIS DOT 14. In an ionic compound A+X–, the degree of covalent bonding
STRUCTURE is greater when
(a) A+ and X– ions are small
1. Metal ‘M’ forms a peroxide of the type MO2. Valency of the
(b) A+ is small and X– is large
metal with respect to oxygen
(c) A+ and X– ions are approximately of the same size
(a) 0 (b) 1 (c) 2 (d) 4
(d) X- is small and A+ is large
2. Valency of the metal atom with respect to oxygen is maximum in
15. Which of the following has highest ionic character?
(a) Mn2O7 (b) OsO4
(a) MgCl2 (b) CaCl2
(c) MnO2 (d) CrO3
(c) BaCl2 (d) BeCl2
3. Variable valency is a property of
16. In which of the following molecule, the central atom has
(a) Alkali metals (b) Transition metals
three lone pairs of electrons ?
(c) Alkaline earth metals (d) Inert gases
(a) Ammonia (b) Xenon difluoride
4. The molecule that deviates from octet rule is (c) Chlorine trifluoride (d) Hydrogen sulphide
(a) NaCl (b) BeCl2 17. Number of lone pairs of electrons in 9 gms. of water are
(c) MgO (d) NH3 [N = Avogadro Number]
5. Which of the following molecule deviates from octet rule (a) 2N (b) N / 2
with respect to central atom? (c) N (d) N / 4
(a) PCl3 (b) H2S (c) NH3 (d) XeF4 18. The number of electron pairs involved in the formation of
6. Expanded octet can be observed in the valency shell of the hydrogen cyanide molecule are
central atom in (a) Two (b) Eight
(a) NH3 (b) CH4 (c) PCl5 (d) BeCl2 (c) Three (d) Four
7. How many number of species are hypovalent ? 19. The number of π-bonds in the structure given below are:
BeCl2, BCl3, CCl4, PCl3, O2, H2 (NC)2 C = C (CN)2
(a) 2 (b) 3 (c) 4 (d) 5 (a) 9 (b) 7 (c) 5 (d) 2
8. Which of the following species will be the strongest Lewis 20. The molecule MLx is planar with six pair of electrons around
acid ? M in the valence shell. The value of x is
(a) Fe0 (b) Fe3+ (c) Fe2+ (d) Fe+ (a) 6 (b) 2 (c) 4 (d) 3
9. Which of following molecule/specie is having maximum 21. The electronegativities of two elements are 0.7 and 3.0. The
number of lone pairs in Lewis - dot structure ? bond formed between them would be
(a) BH −4 (b) BF4− (a) Ionic (b) Covalent
(c) Co-ordinate covalent (d) Metallic
(c) CN¯ (d) COCl2
22. Highly ionic compound is formed by the combination of
10. Which of the following species follows the octet rule for all
elements belonging to
of its atoms ?
(a) I A group and VII A gruop
(a) BeH2‑dimer (b) TeF6
(b) II A group VI A group
(c) XeF6 (d) C2F6
(c) III A group and V A group
11. What is the formal charge on O-atom in carbonate ion.
(d) ‘O’ group and VII A group
(a) –1 (b) 0
(c) +1 (d) – 1 & 0 both 23. An electrovalent compound is made up of
(a) Electrically charged particles
IONIC OR ELECTROVALENT/COVALENT BOND (b) Neutral molecules
12. Molecule having maximum number of covalent bonds is (c) Neutral atoms
(a) NH4OH (b) NH4Cl (d) Electrically charged atom or group of atoms
(c) CO(NH2)2 (d) CH3OH 24. Potassium forms a highly ionic compound when it combines
13. Number of bonded electrons in ethane molecule are with
(a) 7 (b) 12 (a) Chlorine (b) Fluorine
(c) 10 (d) 14 (c) Bromine (d) Iodine
Chemical Bonding and Molecular Structure 83
25. The number of sp2- sigma bonds in benzene are: 38. Which of the following species has bond angle less than
(a) 3 (b) 6 98°?
(c) 12 (d) None of these (a) H2O (b) NH3
(c) CH4 (d) PH3
26. The strength of the bonds by 2s-2s, 2p-2p and 2p-2s overlap
has the order: 39. Find out the relation between (adjacent angle)
(a) s-s > p-p > p-s (b) s-s > p-s > p-p  and ÐFBrF
ÐFClF  bond angle in ClF and BrF molecule
3 3
(c) p-p > p-s > s-s (d) p-p > s-s > p-s respectively.
 = FBrF
(a) FClF   > FBrF
(b) FClF 
27. Most ionic compound among the following is
(a) Sodium fluoride (b) Sodium Chloride  < FBrF
(c) FClF  (d) Can’t predicted
(c) Sodium bromide (d) Sodium iodide 40. The number of P–P–P angle in P4 structure is:
28. Which of the following has least polarity in bond? (a) 6 (b) 9
(a) H – F (b) H – Cl (c) 12 (d) 10
(c) H – O (d) H – S 41. The bond angle and hybridization in ether (CH3OCH3) is :
29. Which contains both polar and non-polar bonds ? (a) 106o51’, sp3 (b) 104o31’, sp3
(a) NH4Cl (b) HCN (c) > 109o 28’ sp3 (d) None of these
(c) H2O2 (d) CH4 42. The bond angle in H2S is:
30. Which of the following is non-polar? (a) > NH3 (b) Same as in BeCl2
(a) H2S (b) NaCl (c) > H2Se, < H2O (d) Same as in CH4
(c) Cl2 (d) H2SO4 43. The smallest bond angle around the central atom will be
31. Which of the following unit conversion of dipole moment is present in:
correct? (a) H2O (b) BeF2
(a) 1D = 3.3356 Cm (c) CH4 (d) NH3
(b) 1D= 3.3365 × 10–30 Cm 44. The bond length of the S–O bond is maximum in which of
(c) 1 C m = 3.3356 D the following compound ?
(d) 1 C m = 3.3356 × 10–30 D SOBr2, SOCl2, SOF2
32. The compound which does not contain ionic bond is: (a) SOCl2 (b) SOBr2
(a) NaOH (b) HCl (c) SOF2 (d) All have same length
(c) K2S (d) LiH 45. Which of the angle is most influenced by the presence of
33. Which of the following chloride has considerable covalent lone pair in SF4 ?
character? (a) Equatorial angle
(a) LiCl (b) NaCl (b) Axial angle
(c) KCl (d) CsCl (c) Angle between axial and equatorial
(d) None
34. Which of the following hydrocarbons has the lowest dipole
moment? 46. Correct order of bond angle in CH2F2:
CH3 H
ˆ H > HCF
(a) HC ˆ > FCF
ˆ ˆ > HCF
(b) FCF ˆ > HCH
ˆ
(a) C=C (b) CH3C ≡ CCH3 ˆ > FCFˆ > HCF ˆ ˆ > FCF ˆ > HCH ˆ
H CH3 (c) HCH (d) HCF
(c) CH3CH2C ≡ CH (d) CH2 = CH – C ≡ CH 47. Which of the following molecule has shortest C-H bond ?
(a) Ethene (b) Ethane
BOND PARAMETERS (c) Ethyne (d) Methane
35. Which of the following is correct order of bond length ?
48. The correct sequence of decrease in the bond angles of the
(a) BF4– < BF3 (b) NO2+ < NO2– following hydrides is:
(c) CCl4 < CF4 (d) +CH3 > CH4 (a) NH3 > PH3 > AsH3 > SbH3
36. Indicate the nature of bonding in diamond. (b) NH3 > AsH3 > PH3 > SbH3
(a) Ionic (b) Covalent (c) SbH3 > AsH3 > PH3 > NH3
­
(c) Molecular (d) Metallic (d) PH3 > NH3 > AsH3 > SbH3
37. In which of the following species, the angle around the 49. Find the maximum number of identical angle in CH2F2
central atom is exactly equal to 109°28ʹ? molecule.
(a) SF4 (b) NH3 (a) 2 (b) 4
(c) NH+4 (d) None of these (c) 6 (d) 8
84 Dropper NEET
50. In which of the following bond angle is maximum: 63. The species/molecule is having same shape and different
(a) NH3 (b) NH4+ hybridisation:
(c) PCl3 (d) SCl2 (a) XeF2, CO2 (b) I3–, H3O+

(c) OCl , CO (d) SO2, OCl2
51. Which of the following H-bonds is expected to have
maximum strength? 64. Which of the following tetra atomic species is non-polar and
(a) H-O----H (b) H-N----H planar?
(c) H-S-----H (d) All have same strength (a) BrF4– (b) NH4+
(c) ClF3 (d) BF3
52. Which one of the following compounds has the smallest
bond angle in its molecule? 65. Which of the possible molecule / species is having maximum
(a) OH2 (b) SH2 (c) NH3 (d) SO2 values for dipole moment. (where “A” is the central atom)?
(a) AX3 (having one lone pair on central atom)
53. In O3, there are:
(b) AX4 (Tetrahedral)
(a) 2σ, 1π bond (b) 1σ, 2π bonds
(c) AX4Y (having no lone pair on central atom)
(c) 2σ, 2π bonds (d) 2σ, 1π bond
(d) Can’t be predicted
54. Which one of the following has the smallest bond angle?
66. Which is the right structure of XeF4 ?
(a) NH3 (b) BeF2
(c) H2O (d) CH4
55. The correct order of bond angles (smallest first) in , H2S,
NH3, BF3 and SiH4 is:
(a) (b)
(a) H2S < NH3 < SiH4 < BF3
(b) NH3 < H2S < SiH4 < BF3
(c) H2S < SiH4 < NH3 < BF3
(d) H2S < NH3 < BF3 < SiH4
56. Which one of the following bonds has the highest average
bond energy? (c) (d)
(a) S = O (b) C ≡ C
(c) C ≡ N (d) N ≡ N
67. The shape of SO42 – ion is:
57. The correct decreasing order of bond angles is:
(a) Square planar (b) Square Pyramidal
(a) ClF3 > PF3 > NF3 > BF3
(c) Tetrahedral (d) None of these
(b) BF3 > PF3 > NF3 > ClF3
(c) BF3 > NF3 > PF3 > ClF3 68. Which of the following species given below have shape
similar to XeOF4 ?
(d) BF3 > ClF3 > PF3 > NF3
(a) XeO3 (b) ΙOF4+
58. Bond angle in water is:
(c) PCl5 (d) XeF5+
(a) 120° (b) 109.5°
(c) 107° (d) 104.5° 69. The total number of lone pairs of electrons in NO3– :
(a) 1 (b) 2
59. The correct order in which the O–O bond length increases in
(c) 6 (d) 8
the following :
(a) H2O2 < O2 < O3 (b) O3 < H2O2 < O2 70. The central atom in IF7 has seven pairs of valency electrons.
The shape of the molecule as per VSEPR theory is:
(c) O2 < H2O2 < O3 (d) O2 < O3 < H2O2
(a) Trigonal bipyramid
VSEPR THEORY (b) Hexagonal pyramid
60. Cyanogen, (CN)2, has a ___ shape/structure: (c) Pentagonal bipyramid
(a) Linear (b) Zig-zag (d) Square bipyramid
(c) V‑shape (d) Cyclic 71. The correct order of decreasing polarity is:
61. The shape of covalent molecule AX3 is: (a) HF > SO2 > H2O > NH3
(a) Triangular (b) T-shape (b) HF > H2O > SO2 > NH3
(c) Pyramidal (d) Any of the above (c) HF > NH3 > SO2 > H2O
62. Of the three molecules XeF4, SF4, SiF4, one which have (d) H2O > NH3 > SO2 > HF
tetrahedral structure is ? 72. Which of the following is a pyramidal species?
(a) All the three (b) Only SiF4 (a) CO2 (b) H3O+
(c) Both SF4 & XeF4 (d) Only SF4 & XeF4 (c) SnCl2 (d) NH4+
Chemical Bonding and Molecular Structure 85
73. In CCl4 the four valencies of carbon are directed towards the VALENCE BOND THEORY & HYBRIDISATION
corners of a
86. Which of the following molecule is having shortest bond
(a) Cube (b) Hexagon length of C–O bond ?
(c) Prism (d) Tetrahedron (a) CH3OH (b) H2CO
74. The shape of CO2 molecule is similar to: (c) CO (d) Na2CO3
(a) H2O (b) BeF2 87. The ratio of σ and π bonds in benzene is:
(c) SO2 (d) None of these (a) 2 (b) 6
75. Out of CHCl3, CH4 and SF4, the molecules having regular (c) 4 (d) 8
geometry are:
88. A hybrid orbital formed from s and p- orbital can contribute to
(a) CHCl3 only (b) CHCl3 and SF4
(a) σ- bond only (b) π-bond only
(c) CH4 only (d) CH4 and SF4
(c) Either σ or π-bond (d) Cannot be predicted
76. In OF2 molecule, the total number of bond pairs and lone
89. The hybrid state of C in CS2 should be:
pairs of electrons are respectively:
(a) sp2 (b) sp
(a) 2, 6 (b) 2, 8
(c) sp3 (d) Not specific
(c) 2, 10 (d) 2, 9
90. Indicate the wrong statement according to Valence bond theory:
77. Which one of the following has the regular tetrahedral structure?
(a) A sigma bond is stronger than π-bond
(a) XeF4 (b) [Ni(CN)4]2–
(b) p-orbitals always have only sidewise overlapping
(c) BF–4 (d) SF4
(c) s-orbitals never form π-bonds
78. The shape of formaldehyde molecule as per the VSEPR (d) There can be only one sigma bond between two atoms
theory is:
91. The strongest P–O bond is found in the molecule
(a) Linear (b) Planar triangle
(a) F3PO (b) Cl3PO
(c) Pyramid (d) Tetrahedron
(c) Br3PO (d) (CH3)3PO
79. Hybrid orbital with least s - character is:
92. In pent-3-en-1-yne the terminal carbon-atoms have following
(a) sp3d (b) sp2
hybridisation:
(c) sp 3 (d) sp
(a) sp & sp2 (b) sp2 & sp3
80. The number of hybrid orbitals in a molecule of decane are: (c) sp2 & sp (d) sp & sp3
(a) 36 (b) 40 93. During the complete combustion of methane CH4, what
(c) 38 (d) 8 change in hybridisation does the carbon atom undergo?
81. The molecule which contains σsp3–sp3 and σsp3–p bonds in it is: (a) sp3 to sp (b) sp3 to sp2
(a) CH3CH3 (c) sp2 to sp (d) sp2 to sp3
(b) CH3CHO 94. Which orbital is used by oxygen atom to form a sigma bond
(c) CH3CH2Cl with other oxygen atom in O2 molecule?
(d) CHCl3 (a) Pure p-orbital (b) sp2 - hybrid orbital
82. Bond angle between two hybrid orbitals is 107o, s - orbital
3
(c) sp - hybrid orbital (d) sp - hybrid orbital
character of hybrid orbital is nearly 95. Which of the following compounds have the same no. of
(a) 50 % (b) 33.33% lone pairs with their central atom ?
(c) 16.6 % (d) 25 %
[I] XeF5− [II] BrF3 [III] XeF2
83. Carbon atoms in C2(CN)4 are: [IV] H3 S + [V] Triplet Methylene
(a) sp hybridized (a) IV and V (b) I and III
(b) sp2 Hybridized (c) I and II (d) II, IV and V
(c) sp and sp2 hybridized 96. The correct order of hybridization of the central atom in the
(d) sp,sp2 and sp3 hybridized following species :
84. The interparticle forces in liquid hydrogen are. NH3, XeO2F2, SeF4, NO2+
(a) H-bonds (a) sp3, sp3, sp3d, sp (b) sp3, sp3d, sp3d, sp
(b) Vander waal's forces 3 3 2 3
(c) sp , sp d , sp d, sp 2 (d) sp2, sp3d, sp3d2, sp
(c) Covalent bonds 97. The hybridisation of the central atom will change when
(d) None of these (a) NH3 combines with H+
85. The number of sigma and Pi bonds in a molecule of cyanogen are: (b) H3BO3 combines with OH–
(a) 4, 3 (b) 3, 4 (c) NH3 forms NH −2
(c) 5, 2 (d) 3, 5 (d) H2O combines with H+
86 Dropper NEET
98. What is the state of hybridisation of anionic part of solid 110. pi bond is formed by the overlapping of
N2O5 ? (a) p - p orbitals along their axis
(a) sp (b) sp2 (b) s - p orbitals along the axis of p - orbital
(c) sp 3 (d) Not applicable (c) p - p orbitals perpendicular to their axis
99. Which one of the following pairs is isostructural? (d) s - s orbitals
(a) [BCl3 and BrCl3] (b) [NH3 and NO3– ] 111. Which of the following is not correct ?
(c) [ NF3 and BF3 ] (d) [BF4- and NH +4 ] (a) A sigma bond is weaker than pi bond
100. The hybridisation of C‑atoms in tetracyanomethane is (b) A sigma bond is stronger than pi bond
(a) sp, sp2 (b) sp3, sp (c) A double bond is stronger than a single bond
3
(c) sp , sp 3 (d) sp3 , sp2 (d) A double bond between two atoms is shorter than a
101. BF3 + F– → BF4– single bond between the same atoms.
What is the hybridiation state of B in BF3 and BF4– ? 112. The s - character in the hybrid orbital of the central atom,
(a) sp2, sp3 (b) sp3, sp3 present in a molecule having the shape of an octahedron is
2
(c) sp , sp 2 (d) sp3, sp3d (a) 25 % (b) 75 %
(c) 40 % (d) 16.66 %
102. Choose the molecules in which hybridisation occurs in the
ground state ? 113. The C-H bond in propane is
(i) BCl3 (ii) NH3 (a) σsp - s (b) σsp2–s (c) σp - s
(d) σsp3–s
(iii)PCl3 (iv) BeF2 114. The hybridisation of P in phosphate ion (PO43 –) is the same

The correct answer is as:
(a) i, ii, iv (b) i, ii, iii (a) Ι in ΙCl4– (b) S in SO3
(c) ii, iii (d) iii, iv (c) N in NO3– (d) S in SO32 –

103. The geometry of electron pairs around I in IF5 is: 115. Number of hybrid orbitals present in a molecule of propene
(a) Octahedral (b) Trigonal Bipyramidal are
(c) Square Pyramidal (d) Pentagonal Planar (a) 12 (b) 10 (c) 9 (d) 8
104. sp3d hybridization is considered to be a combination of two 116. Molecule obtained by sp3d2 hybridisation has bond angle of
hybridization. They are (a) 90o (b) 109o28'
(a) p3 + sd (b) sp2 + pd (c) 120 o
(d) 72o
(c) spd + p2 (d) None of these 117. Hybrid orbital having maximum p - character is
105. All the four sigma bonds in perchlorate ion are formed by (a) sp3d (b) sp3
which orbital? (c) sp (d) sp3d2
(a) sp3d3 (b) sp3d2 118. Hybridisation involves
(c) sp3d (d) sp3 (a) Addition of an electron pair
106. Which one of the following conversions involve change in (b) Combination and redistribution of atomic orbitals
both hybridisation and shape?
(c) Removal of an electron pair
(a) CH4 → C2H6 (b) NH3 → NH+4
(d) Separation of orbitals
(c) BF3 → BF4 – (d) H2O → H3O+
119. On hybridisation of one ‘s’ and one ‘p’ orbitals we get
107. Which of the following penta atomic species is tetrahedral
(a) Two mutually perpendicular orbitals
and non polar ?
+
(b) Two orbitals at 180o
(a) CH3F (b) NH 2 F2 (c) Four orbitals directed tetrahedrally
(c) [Ni(CO)4] (d) BF4− (d) Three orbitals in a plane
108. The orbital overlapping is maximum in 120. As the s - character in a hybrid orbital increases, the bond
(a) Cl2 (b) HI angle
(c) HCl (d) HBr (a) Increases (b) Decreases
(c) Does not change (d) Becomes zero
109. Linear combination of two hybridised orbitals belonging to
two atoms and each having one electron leads to a 121. sp3d hybridisation results in
(a) Sigma bond (a) A square planar molecule
(b) Double bond (b) An octahedron molecule
(c) Co-ordinate covalent bond (c) A trigonal bipyramid molecule
(d) pi bond (d) A tetrahedron molecule

Chemical Bonding and Molecular Structure 87


122. s - p overlapping is present in 134. On catalytic hydrogenation, ethylene gives ethane during
(a) Br2 (b) H2 this reaction
(c) O2 (d) HF (a) Hybridization of carbon atoms changes from sp2 to sp3
123. Valence bond theory of Pauling and Slater accounts for the (b) bond angle decreases from1200 to 109.50
following characteristic of covalent bond (c) C-C bond length increases from1.34 A0 to 1.54A0
(a) Directional (b) ionic (d) All of these.
(c) Strength (d) hybrid 135. The ratio of pure orbitals to hybridized orbitals in ethylene is
124. Shape of molecule is decided by (a) 2 : 3 (b) 3 : 1
(a) Sigma bonds (b) pi bonds (c) 1 : 1 (d) 1 : 3
(c) Both sigma and pi bonds (d) Neither sigma nor pi bonds
MOLECULAR ORBITAL THEORY
125. The strength of bonds by 2s-2s, 2p-2p, 2s-2p, 1s-1s overlap
has the order: 136. Which of the following species has lowest ionization
(a) 1s-1s > 2p-2p > 2s-2p > 2s-2s potential?
2−
(b) 2p-2p > 2s-2p > 2s-2s > 1s-1s (a) O2 (b) O2
(c) 2s-2s > 1s-1s > 2s-2p > 2p-2p (c) O2+ (d) O2–
(d) 2s-2p > 2s-2s > 2p-2p > 1s-1s 137. Among KO2, AlO2– , BaO2 & NO2+ unpaired electron is
126. The hybridization of atomic orbitals of nitrogen in NO2+ , present in:
NO3– and NH4+ are (a) NO2+ and BaO2 (b) KO2 and AlO2–
(a) sp2, sp3 and sp2 respectively (c) KO2 only (d) BaO2 only
(b) sp, sp2 and sp3 respectively 138. What is correct sequence of bond order ?
(c) sp2, sp and sp3 respectively (a) O2+ > O2– > O2 (b) O2+ > O2 > O2–
(d) sp2, sp3 and sp respectively (c) O2 > O2– > O2+ (d) O2– > O2+ > O2
127. According to valence bond theory the predicted bond angle 139. Which statement is correct about O2+ ?
for H2O
(a) Paramagnetic and bond order < O2
(a) 90° (b) 109°28′
(c) 107°18′ (d) 104°28′ (b) Paramagnetic and bond order > O2

128. Which one of the following is a correct set? (c) Diamagnetic and bond order < O2
(a) H2O,sp3,angular (b) H2O,sp2, linear (d) Diamagnetic and bond order > O2
(c) NH4 ,dsp , square planar (d) CH4,dsp2, tetrahedral
+ 2 140. In which of the following ionization processes, the bond
order has increased and the magnetic behaviour has changed?
129. Which one of the following is the correct set with reference (a) C2 → C2+ (b) NO → NO+
to molecular formula, hybridisation of central atom and (c) O2 → O2 + (d) N2 → N2+
shape of the molecule?
141. The molecular electronic configuration of H2– ion is
(a) CO2,sp2, bent (b) H2O, sp2, bent
(a) (σ1s)2 (b) (σ1s)2(σ*1s)2
(c) BeCl2, sp, linear (d) H2O, sp3, linear 2 1
(c) (σ1s) (σ*1s) (d) (σ1s)3
130. What is the hybridisation state of the central atom in the
142. Which one of the following molecules contains no π-bond?
conjugate base of NH4+ ion?
(a) CO2 (b) H2O
(a) sp (b) sp3 (c) SO2 (d) NO2
(c) sp 2 (d) dsp2
143. Which of the following molecules/ions does not contain
131. For which hybridization, there are two unequal bond angles unpaired electrons?
(a) sp3 (b) sp2 (a) O 22 − (b) B2
(c) sp (d) sp3d
(c) N-2 (d) O2
132. The shape of CH4; SO4-2; PO4-3 is
144. Anti- bonding molecular orbital is formed by
(a) Trigonal planar
(a) Addition of wave functions of atomic orbitals
(c) Angular
(b) Substraction of wave functions of atomic orbitals
(c) Tetrahedral
(c) Multiplication of wave functions of atomic orbitals
(d) Trigonal Bipyramidal
(d) Finding the arithmetic mean
133. The compound in which carbon uses sp3 hybridisation for
145. Which one of the following species does not exist under
bond formation
normal conditions?
(a) H-COOH (b) (NH2)2C=O
(a) Li2 (b) Be +2
(c) H-CHO (d) CH3CH2 OH (c) Be2 (d) B2
88 Dropper NEET
146. Oxygen molecule is HYDROGEN BONDING
(a) Diamagnetic with no unpaired electrons 160. Which compound has electrovalent, covalent, coordinate as
(b) Diamagnetic with two unpaired electrons well as hydrogen bond ?
(c) Paramagnetic with two unpaired electrons
(a) NH4Cl (b) HNO3
(d) Paramagnetic with no unpaired electrons
(c) CuSO4.5H2O (d) BeCl2.2H2O
147. The species having bond order different from that in CO is
(a) NO– (b) NO+ 161. Which of the following is the correct order of strength of
– H‑bonding in the given compound ?
(c) CN (d) N2
(a) HF < NH3 (b) H2O > H2O2
148. Which one of the following has the strong O-O bond?
(c) H2O2 > H2O (d) NH3 > H2O
(a) O +2 (b) O2 162. Strongest hydrogen bond is present in
(c) O-2 (d) O 22- (a) Ammonia (b) Water
(c) Hydrogen fluoride (d) Ethyl alcohol
149. In which of the following pairs, the two species have identical
bond order? 163. Intermolecular hydrogen bond is not present in
(a) N-2 , O 22- (b) N-2 , O-2 (a) Ammonia (b) Water
(c) Hydrofluoric acid (d) Salicylaldehyde
(c) N-2 , O+2 (d) O +2 , N 22-
164. Which of the following exists as a liquid at room temperature
due to the formation of associated molecules only?
150. How may bonds does B2 have?
(a) Benzene (b) Ammonia
(a) 0 (b) 1
(c) Bromine (d) Carbon disulphide
(c) 2 (d) 3
165. Which of the following species has intramolecular H-bonds?
151. According to MOT, C2 molecule has
(a) Phenol (b) o-Nitrophenol
(a) 1σ & 1π bond (b) Only 2π-bond
(c) p-Nitrophenol (d) Nitroethane
(c) Only 2σ-bond (d) 1σ and 2π bond
166. Bond energy of covalent O - H bond in water is
152. Which of the following is diamagnetic?
(a) Greater than bond energy of hydrogen bond
(a) H + (b) O2
2 (b) Equal to bond energy of hydrogen bond
(c) Li2 (d) He +2
(c) Less than bond energy of hydrogen bond
153. Molecular orbital theory was given by: (d) Half of the bond energy of hydrogen bond
(a) Kossel (b) Moseley
167. Water is a liquid while hydrogen sulphide is a gas because
(c) Mulliken (d) Werner
(a) Water has higher molecular weight
154. Molecular orbital electronic configuration for ʻXʼ anion is- (b) Hydrogen sulphide is a weak acid
KK σ*2s2 σ*2s2 π 2px2 π 2py2 σ2pz2 π* 2px1, the anion ʻXʼ is
(c) Sulphur has high electronegativity than oxygen
(a) N-2 (b) O-2 (d) Water molecules associate through hydrogen bonding
(c) N 22- (d) O 22 − 168. Which of the following is a normal liquid ?
155. The number of nodal planes present in σ*2s anti- bonding (a) NH3 (b) H2O
orbital is (c) HF (d) Br2
(a) 1 (b) 2 169. Among the following, the boiling point is high for
(c) 0 (d) 3 (a) Ethyl alcohol (b) Dimethyl ether
156. Which of the following is paramagnetic with bond order 0.5? (c) Acetone (d) Chloroform
(a) F2 (b) H +2 170. Strongest hydrogen bond is present in
(a) CH3OH (b) CHCl3
(c) N2 (d) O-2
(c) (CH3)2CHOH (d) (CH3)3COH
157. Which of the following pairs have identical bond order?
171. Which of the following is miscible with water ?
(a) N 2 , O 22 + (b) N 2 , O 2−
(a) CS2 (b) C2H5OH
(c) N −2 , O 2 (d) O 2 + , N 2 (c) CCl4 (d) CHCl3

158. The bond order of O2 is: 172. The coupling between base units of DNA is through
(a) 0.5 (b) 1.5 (a) Hydrogen bonding (b) Electrostatic bonding
(c) 3.5 (d) 2.5 (c) Covalent bonding (d) Vander Waal’s forces
159. The molecule having bond order 3 is 173. Which of the following is steam volatile ?
(a) H2 (b) N2 (a) Phenol (b) o - Nitrophenol
(c) O2 (d) He2+ (c) m - Nitrophenol (d) p - Nitrophenol
Chemical Bonding and Molecular Structure 89
174. Which of the following compounds would show evidence of 184. H-bond is not present in:
the strongest hydrogen bonding ? (a) Water
(a) Propan-l-ol (b) Propan-2-ol (b) Glycerol
(c) Propan-l,2-diol (d) Propan-l,2,3-triol (c) Hydrogen fluoride
175. The compound having hydrogen bonds in it is (d) Hydrogen sulphide
(a) NH3 (b) H2S 185. Which of the following has strongest hydrogen bonding?
(c) HCl (d) PH3 (a) Ethyl amine (b) Ammonia
176. Strength of hydrogen bond is intermediate between (c) Ethyl alcohol (d) Diethyl Ether
(a) Vander Waal forces and covalent bond 186. The high boiling point of water is due to-
(b) Ionic bond and covalent bond (a) Co-ordinate bonding
(c) Ionic bond and metallic bond (b) Covalent bonding
(d) Resonance (c) Electrostatic force of attraction
(d) Hydrogen Bonding
177. NH3 has a much higher boiling point than PH3 because
(a) NH3 has a larger molecular weight 187. When two ice cubes are pressed over each other, they unite to
form one cube. Which of the following forces is responsible
(b) NH3 undergoes umbrella inversion
to hold them together?
(c) NH3 contains hydrogen bonds
(a) Dipole forces
(d) NH3 contains ionic bonds where as PH3 contains covalent
(b) Vander waal's forces
bonds
(c) Covalent forces
178. The maximum number of molecules that one water molecule
(d) Hydrogen bond forces
can hold through hydrogen bonding is
(a) 2 (b) 4 188. Hydrogen bonding is maximum in:
(c) 6 (d) 8 (a) Ethyl chloride (b) Triethyl amine
(c) Ethanol (d) Diethyl ether
179. The high density of water compared to ice is due to
(a) H-bonding interactions 189. What is the dominant intermolecular force or bond that must
(b) Dipole-dipole interactions be overcome in converting liquid CH3OH to a gas?
(c) Dipole-induced dipole interactions (a) Hydrogen bonding
(d) Induced dipole-induced dipole interactions (b) Dipole-dipole force
(c) Covalent bond
180. The force responsible for the union of two ice blocks as a
single block is (d) London dispersion force
(a) Vanderwaals force 190. Which of the following is least volatile?
(b) Hydrogen bonds (a) H2O (b) H2S
(c) dipole interaction (c) H2Se (d) H2Te
(d) vanderwaals repulsion 191. The hydrogen bonding is maximum strong in:
181. The Electronegative elements between which the H-bond is (a) S-H-----S (b) N-H-----O
formed principally in aqueous ammonia (c) S-H-----O (d) F-H------O
(a) O, O (b) N,N 192. The comparatively high boiling point of HF is due to
(c) O, N (d) Both O,O & N,N (a) High reactivity of fluorine
182. Which of the following form hydrogen bond ? (b) Small size of hydrogen atom
(a) CH3F (b) CH3Cl (c) Formation of hydrogen bonds and consequent association
(c) CH3Br (d) None (d) High IE of fluorine
183. Chelation is observed in 193. Intramolecular H-bonding is present in:
(a) P-nitrophenol (b) O-nitrophenol (a) Meta- nitrophenol (b) Salicyaldehyde
(c) Benzaldehyde (d) HF (c) HCl (d) Benzophenone

90 Dropper NEET
Learning Plus
1. The correct order of hybridization of the central atom in the 8. Among the triatomic molecules/ions BeCl2 , XeF2, ICl2+,
following species NH3, PCl5 and BCl3 is I3+, I(CN)2–
(a) sp3d, sp2 and sp3 (b) sp3 , sp3d, sp2 How many are sp3d hybridised and linear choose the
correct pairs ?
(c) sp2, sp3, sp3d (d) sp3, sp2, sp3d
(a) BeCl2 & XeF2 (b) XeF2 & I(CN)2–
2. How many molecule which has sp3d hybridisation but planar
as well as non-polar in structure? (c) XeF2 & ICl2+ (d) BeCl2 & I3+

[ICl2]–, BrF5, SOF4, SF6, XeOF4 , ClF3, I3 , [I(CN)2]–, PCl5 9. In PO43– ion the average formal charge on the oxygen
(a) 2 (b) 4 atom of P–O bond is:
(c) 6 (d) 8 (a) + 1 (b) –1
(c) – 0.75 (d) + 0.75
3. Polarity in a molecule and hence the dipole moment –
depends primarily on electronegativity of the constituent 10. In NO3 ion, the number of bond pairs and lone pairs of
atoms and shape of a molecule. Which of the following electrons on nitrogen atom are:
has the highest dipole moment? (a) 2, 2 (b) 3, 1
(c) 1, 3 (d) 4, 0
(a) CO2 (b) HI
(c) H2O (d) SO2 11. If the equatorial plane is x–y plane in sp3d hybridisation
then the orbital used in pd hybridisation are -
4. Choose the INCORRECT options.
(a) pz and dz2 (b) px and dxy
(a) All dN-O bond lengths are equal in nitrate ion.
(c) py and dyz (d) None of these
(b) All dC-O bond lengths are equal in sodium carbonate.
12. How many maximum number of atoms are present in one
(c) All dC-O bond lengths are equal in acetic acid. plane in PF2(CH3)3 molecule, only consider those geometry
(d) All dB-O bond length are equal in boric acid. which is non-polar in nature?
5. In a compound

The number of sigma and pi bonds respectively are


(a) 19, 11 (b) 19, 5 (a) 2 (b) 4 (c) 7 (d) 8
(c) 13, 11 (d) 7, 3 13. Which of the following statement is true for IO2F2– ?
6. Hydrogen bonds are formed in many compounds e.g., H2O, (a) The electrons are located at the corners of a trigonal
HF, NH3. The boiling point of such compounds depends to a bipyramidal but one of the equatorial pairs is unshared.
extent on the strength of hydrogen bond and the number of (b) It has sp3d hybridisation and is T–shaped.
hydrogen bonds. The correct decreasing order of the boiling (c) Its structure is analogous to SF4.
points above compounds is: (d) (a) and (c) both
(a) HF > H2O > NH3 (b) H2O > HF > NH3 14. Which of the following species has tetrahedral geometry?
– –
(c) NH3 > HF > H2O (d) NH3 > H2O > HF (a) BH4 (b) NH2
7. Choose the correct statement: (c) CO32 – (d) H3O+
(a) NH3 is having bond angle of 109°28’. 15. Number of π bonds and σ bonds in the following structure
(b) The direction of the dipole moment of NF3 is as shown is:
in the diagram

(c) sp2 hybrid orbital is consisting of 66.67 % ‘p’ character.


(d) None of these (a) 6, 19 (b) 4, 20 (c) 5, 19 (d) 5, 20

Chemical Bonding and Molecular Structure 91


16. In NO2+, N undergoes 27. Which of the following angle corresponds to sp2
(a) sp3 hybridisation (b) sp2 hybridisation hybridization?
(c) sp hybridisation (d) sp2d hybridisation (a) 90° (b) 120°
17. Which of the following should have pyramidal shape ? (c) 180° (d) 109°
(a) [ClOF2]+ (b) ICl3 28. The bond angle in PH3 is :
(c) [BrΙCl]– (d) All of these (a) Much lesser than NH3
18. Among the triatomic molecules/ion BeCl2 , NO2+, ICl2–, XeF2 (b) Equal to that in NH3
, I3–, OCl2 total number of linear molecule(s)/ion (s) are : (c) Much greater than in NH3
(a) 2 (b) 3 (d) Slightly more than in NH3
(c) 4 (d) 5 29. In which of the following sets, all the species are diamagnetic
19. Which molecule/ion out of the following does not contain in nature?
unpaired electrons? (a) [O22–, O22+ , N22– ] (b) [C2, O22–, N22+]
(a) N2+ (b) O2 (c) O22 – (d) B2 (c) [O22–, O2+, O22–] (d) [N2+, O22+, B2]
20. In which of the following molecule/ion all the bonds are not 30. Which of the following order of energies of molecular
equal? orbitals of N2 is correct?

(a) XeF4 (b) BF4 (c) C2H4 (d) SiF4
(a) ( π2p y ) < ( σ2p z ) < ( π* 2p x ) ≈ ( π* 2p y )
21. The geometry of ammonia molecule can be best described
as: (b) ( π2p y ) > ( σ2p z ) > ( π* 2p x ) ≈ ( π* 2p y )
(a) Nitrogen at one vertex of a regular tetrahedron, the other
(c) ( π2p y ) < ( σ2p z ) > ( π 2p x ) ≈ ( π 2p y )
* *
three vertices being occupied by the three hydrogens
(b) Nitrogen at the centre of the tetrahedron, three of the
(d) ( π2p y ) > ( σ2p z ) < ( π* 2p x ) ≈ ( π* 2p y )
vertices being occupied by three hydrogens
(c) Nitrogen at the centre of an equilateral triangle, three 31. Which of the following statement is not correct from the
corners being occupied by three hydrogens view point molecular orbital theory?
(d) Nitrogen at the junction of a T, three open ends being (a) Be2 is not a stable molecule
occupied by three hydrogens (b) He2 is not stable but He2+ is expected to exist.
22. Which molecular geometry is least likely to result from a (c) Bond strength of N2 is maximum amongst the
trigonal bypyramidal electron geometry? homonuclear diatomic molecules belonging to the
(a) Trigonal planar (b) see–saw second period.
(c) Linear (d) T–shaped (d) The order of energies of molecular orbitals in N2
2 2 2 2 2 2
molecule is σ1s σ *1s , σ2s , σ * 2s , σ2p z , π2p x =π2p 2y
23. Which of the following has bond angle greater than 120°?
32. Among KO2, Al2O3 and BaO2, the unpaired electron is
(a) OCl2 (b) O(CH3)2
present in:
(c) O(SiH3)2 (d) NO2–
(a) KO2 only (b) KO2, BaO2
24. The correct order of bond angle is :
(c) Only Al2O3 (d) Only BaO2
(a) H2S < NH3 < BF3 < CH4
33. The fluoride whose value of dipole moment is not equal to
(b) NH3 < H2S < CH4 < BF3
zero, is:
(c) H2S < NH3 < CH4 < BF3
(a) XeF4 (b) CF4
(d) H2S < CH4 < NH3 < BF3
(c) SF4 (d) PF5
25. Consider the following molecules ;
34. In which of the following, d-orbitals are not used by central
H2O H2S H2Se H2 Te atom in hybridisation?
I ΙΙ ΙΙΙ ΙV (a) PF5 (b) PCl5
Arrange these molecules in increasing order of bond angles. (c) PBr5 (gaseous) (d) None of these
(a) Ι < ΙΙ < ΙΙΙ < ΙV (b) IV < III < II < I 35. BrO3− is isostructural with which of the following ?
(c) Ι < ΙΙ < ΙV < ΙΙΙ (d) ΙΙ < ΙV < ΙΙΙ < Ι
(a) XeO3 (b) XeF3
26. If the electronic configuration of an element is 1s2 2s2 2p6
3s2 3p6 3d2 4s2, the four electrons involved in chemical bond (c) XeF4 (d) XeO2
formation will be: 36. In which of the following compounds B–F length is shortest ?
(a) 3p6 (b) 3p6, 4s2 (a) BF4– (b) BF3 → NH3
(c) 3p6, 3d2 (d) 3d2, 4s2 (c) BF3 (d) BF3 → N(CH3)3

92 Dropper NEET
Multiconcept MCQs
1. Assuming the bond direction to be z‑axis, which of the 6. Choose the correct order of bond angle.
overlapping of atomic orbitals of two atom (A) and (B) will
Column-I Column-II
result in bonding?
A NO2+ 1. 180°
(I) s‑orbital of A and px orbital of B
B NO2 2. 134°
(II) s‑orbital of A and pz orbital of B
C NO2– 3. 120°
(III) py‑orbital of A and pz orbital of B
D NO3– 4. 115°
(IV) s‑orbitals of both (A) and (B)
(a) I and IV (b) I and II (c) III and IV (d) II and IV 5. 109°

2. Choose the INCORRECT property for the following molecules: A B C D A B C D


(I) CH2F2 ; (II) CHF3 ; (III) CH3F (a) 5 4 3 2 (b) 5 2 4 3
(a) C–F bond length order : CH3F > CH2F2 > CHF3 (c) 1 2 4 3 (d) 1 4 3 2

(b) C–H bond length order : CH3F > CH2F2 > CHF3 7. N2 and O2 are converted to monocations N2+ and O2+respectively,
which is wrong statement ?
(c) Shape of the all species is perfectly tetrahedral
(a) In N2+ , the N–N bond weakens
(d) Dipole moment is non zero for given compounds. (b) In O2+, the O–O bond order increases
(c) In O2+, the paramagnetism decreases
3. Identify the correct match.
(d) N2+ becomes diamagnetic
(i) XeF2 (A) Central atom has sp3 hybridisation
and bent geometry. 8. Give the correct order of initials T or F for following
statements. Use T if statement is true and F if it is false.
(ii) N3– (B) Central atom has sp3d2 hybridisation (i) The order of repulsion between different pair of electrons
and octahedral. is lp – lp > lp – bp > bp – bp
(iii) PCl6–(PCl5 (s) (C) Central atom has sp hybridisation (ii) In general, as the number of lone pair of electrons on
anion) and linear geometry. central atom increases, value of bond angle from normal
3 bond angle also increases.
(iv) ICl2+ (I2Cl6 (l) (D) Central atom has sp d hybridisation
cation) and linear geometry. (iii)The number of lone pair on O in H2O is 2 while on N in
NH3 is 1.
(a) (i – A), (ii – B), (iii – C), (iv – D) (iv) The structures of Xenon‑fluorides and Xenon‑oxyfluorides
could not be explained on the basis of VSEPR theory.
(b) (i – D), (ii – B), (iii – D), (iv – C)
(a) TTTF (b) TFTF
(c) (i – B), (ii – C), (iii – A), (iv – D)
(c) TFTT (d) TFFF
(d) (i – D), (ii – C), (iii – B), (iv – A)
9. The H bond in solid HF can be best represented as:
4. Which is not correctly matched?
(a) H–F...H–F...H–F
(a) XeO3 ; Trigonal bipyramidal
(b)
(b) ClF3 ; bent T-shape
(c) XeOF4 ; Square pyramidal
(c)
(d) XeF2 ; Linear shape
5. Which of the following compounds have the same no. of
lone pairs with their central atom ? (d)
[I] XeF5– ; [II] BrF3 ; [III] XeF2 ; [IV] H3S+ [V] : CH2
10. The experimental value of the dipole moment of HCl is 1.03
(a) IV and III D. The length of the H – Cl bond is 1.278Å. The percentage
(b) I and III of ionic character in HCl is:
(c) I and II (a) 43 (b) 21
(d) II, IV and V (c) 17 (d) 7

Chemical Bonding and Molecular Structure 93


11. Which of the following pair of molecules have the same 19. Match List I (Molecules)with List II (Bond order) and select
shape but difference in polarity (polar or non-polar)? the correct answer using the codes.
(a) H2O and NH3 (b) SnCl2 and SO2 List – I List – II
(c) CO2 and N2O (d) SO2 and SO3 I. Li2 A. 3
II. N2 B. 1.5
12. Hydrogen bonding plays a central role in the following
phenomenon : III. Be2 C. 0
(a) Ice floats in water IV. O2 D. 0
(b) Higher Lewis basicity of primary amines than tertiary E. 2
amines in aqueous solutions. Codes
(c) Formic acid is more acidic than acetic acid (a) I - B, II - C, III - A, IV - E (b) I - C, II - A, III - D, IV - E
(d) Dimerisation of acetic acid in benzene (c) I - D, II - A, III - E, IV - C (d) I - C, II - B, III - E, IV - A
13. Among the following pairs, which two species are not 20. The Atomic numbers of three elements A,B and C are a, a+1
isostructural? and a+2. C is an alkali metal. In a compound of A and C, the
nature of bonding is
(a) PF6– and SF6 (b) SiF4 and SF4
(a) Coordinate
(c) IO3– and XeO3 (d) BH4– , NH4+ (b) Covalent
14. Which compound exhibits maximum dipole moment among (c) Ionic
the following? (d) Metallic
21. The electronegativity difference between N and F is greater
than N and H, yet the dipole moment of NH3 (1.5D) is
(a) (b) greater than that of NF3 (0.2D). This is because:
(a) In NH3 as well as NF3, the orbital dipole and bond dipole
are in opposite direction
(b) In NH3 , the orbital dipole and bond dipole are in the opposite
(c) (d) direction, where as in NF3 these are in same direction
(c) In NH3 , as well as in NF3 the orbital dipole and bond
dipole are in same direction.
(d) In NH3, the orbital dipole and bond dipole are in same
15. Which of the following is a wrong order with respect to the direction where as in NF3 these are in opposite direction.
property mentioned against each ?
22. Which of the following statements are correct?
(a) O22– > O2 > O2+ [Paramagnetic moment] (I) The hybridisation found in cation of solid PCl5 is sp3.
(b) Bond Angle : H2O > H2S > H2Se (II) In AB2L2 type, the BAB bond angle is always greater than
(c) H2 > H2+ > He2+ [stability] the normal tetrahedral bond angle.
(d) NO2+ > NO2 > NO2– [bond angle] (III) In ClO3–, NH3 and XeO3, the hybridisation and the
number of lone pairs on the central atoms are same.
16. O2F2 is an unstable yellow orange solid and H2O2 is a
(IV) In discrete P4 molecule, there are six P—P bonds and
colourless liquid, both have O–O bond and O–O bond length
four lone pairs of electrons.
in H2O2 and O2F2 respectively is -
(a) I, II and III only
(a) 1.22 Å, 1.48 Å
(b) I, III and IV only
(b) 1.48 Å, 1.22 Å (c) III and IV only
(c) 1.22 Å, 1.22 Å (d) All are correct
(d) 1.48 Å, 1.48 Å 23. Which of the following statements are correct? (As the
17. Which bond angle θ would result in maximum dipole s-character of a hybrid orbital decreases).
moment for the triatomic molecule XY2 shown below ? (I) The bond angle decreases
(a) θ = 90° (b) θ = 120° (II) The bond strength increases
(c) θ = 150° (d) θ = 180° (III) The bond length increases.
18. Among the following select the correct statements: (a) (I) and (III) (b) (II) and (III)
(c) (I) and (II) (d) All are correct
I. PH6 does not exist
24. The correct order of increasing C - O bond length of CO,
II. pπ-dπ bond is present in SO2
CO32 – , CO2 is
III. SeF4 and CH4 have the same shape (a) CO32 – < CO2 < CO
IV. I3+ has a bent shape (b) CO2 < CO32 – < CO
(a) II, III (b) I, II (c) CO < CO32 – < CO2
(c) I, IV (d) I, II, IV (d) CO < CO2 < CO32 –
94 Dropper NEET
NEET Past 10 Years Questions

1. BF3 is planar and electron deficient compound. Hybridization The bond energy of H2 is  (2020 Covid Re-NEET)
and number of electrons around the central atom, (c− a) (b − a)
respectively are: (2021)
(a) (b)
2 2
(a) sp3 and 6 (b) sp2 and 6 (c) (c – a) (d) (b – a)
(c) sp2 and 8 (d) sp3 and 4
8. Identify the wrongly matched pair. (2020 Covid Re-NEET)
2. Match List-I with List-II. (2021)
Molecule Shape or geometry of molecule
List-I List-II
(a) SF6 Octahedral
(A) PCl5 (i) Square pyramidal
(B) SF6 (ii) Trigonal planar (b) BeCl2 Linear
(C) BrF5 (iii) Octahedral (c) NH3 Trigonal pyramidal
(D) BF3 (iv) Trigonal bipyramidal (d) PCl5 Trigonal planar
Choose the correct answer from the options given below.
9. Which of the following diatomic molecular species has only
(a) A-ii B-iii C-iv D-i
p bonds according to Molecular Orbital Theory?  (2019)
(b) A-iii B-i C-iv D-ii
(a) O2 (b) N2
(c) A-iv B-iii C-ii D-i
(c) C2 (d) Be2
(d) A-iv B-iii C-i D-ii
10. Identify the incorrect statement related to PCl5 from the
3. Which of the following molecules is non-polar in nature?(2021) following:  (2019)

(a) CH2O (b) SbCl5 (a) Three equatorial P-Cl bonds make an angle of 120° with
each other
(c) NO2 (d) POCl3
(b) Two axial P-Cl bonds make an angle of 180° with each
4. Which of the following set of molecules will have zero other
dipole moment?  (2020)
(c) Axial P-Cl bonds are longer than equatorial P-Cl bonds
(a) Boron trifluoride, hydrogen fluoride, carbon dioxide, (d) PCl5 molecule is non-reactive
1,3-dichlorobenzene
11. Consider the following species: (2018)
(b) Nitrogen trifluoride, beryllium difluoride, water 1,
CN+, CN–, NO and CN
3-dichlorobenzzene
Which one of these will have the highest bond order?
(c) Boron trifluoride, berylium difluoride, carbon dioxide,
(a) NO (b) CN– (c) CN (d) CN+
1,4-dichlorobenzene
(d) Ammonia, beryllium difluoride, water, 12. Which one of the following pair of species have the same
1,4-dichlorobenzene bond order? (2017-Delhi)
(a) N 2 ,O 2

(b) CO, NO
5. Identify a molecule which does not exist.  (2020)
(a) Li2 (b) C2 (c) O2 (d) He2 (c) O2, NO+ (d) CN–, CO
6. How many (i) sp2 hybridised carbon atoms and (ii) π bonds 13. The species, having bond angles of 120° is (2017-Delhi)
are present in the following compound? (2020 Covid Re-NEET) (a) BCl3 (b) PH3
(c) ClF3 (d) NCl3
14. Which one of the following ions is not tetrahedral in shape?
(a) 8, 6 (b) 7, 6 (c) 8, 5 (d) 7, 5  (2017-Gujarat)
(a) [NiCl4] 2– (b) NH 4+
7. The potential energy (y) curve for H2 formation as a function
of internuclear distance (x) of the H atoms is shown below. (c) BF4− (d) [Cu(NH3)4]2+
15. Which of the following pair of species is not iso-structural?
 (2017-Gujarat)
(a) BrO3 ,XeO3

(b) ICl4 ,XeF4

(c) ClO3− ,CO32 − (d) IBr2− ,XeF2

Chemical Bonding and Molecular Structure 95



16. Which of the following hydrides has the largest bond angle? 24. Decreasing order of stability of O2 , O2 , O2+ and O22– is:
 (2017-Gujarat)  (2015 RE)
(a) H2Se (b) H2S (c) H2Te (d) H2O –
(a) O2– > O2– > O2+ > O2 (b) O2+ > O2 > O2 > O22–
2
17. Which one of the following compounds shows the presence – –
(c) O2– > O2 > O2 > O2+ (d) O2 > O2+ > O22– > O2
of intramolecular hydrogen bond? (2016 - II) 2

(a) Cellulose (b) Concentrated acetic acid 25. The total number of π-bond electrons in the following structure
is: (2015)
(c) H2O2 (d) HCN
18. In which of the following molecules, all atoms are coplanar?
 (2016 - II)

(a) (b)
(a) 8 (b) 12 (c) 16 (d) 4
26. Which of the following options represents the correct bond
order? (2015)
(a) O2– < O2 < O+ (b) O2– > O2 < O +
(c) (d) 2 2
(c) O2– < O2 > O2+ (d) O2– > O2 > O2+
27. The correct bond order in the following species is: (2015)
– –
(a) O2+ < O2 < O2+ (b) O2+ < O2 < O2+
2 2
19. Among the following which one is a wrong statement? (2016 - II) – –
(c) O2 < +
O2 < 2+
O2 (d) 2+
O2 < O2+< O2
(a) SeF4 and CH4 have same shape
(b) I3+ has bent geometry 28. Which of the following pairs of ions are isoelectronic and
isostructural? (2015)
(c) PH5 and BiCl5 do not exist
(a) SO3 2– , NO3 – –
(b) ClO3 , SO3 2–
(d) p� - d� bonds are present in SO2
2–
(c) CO3 , SO3 2– (d) ClO3– , CO32–
20. The hybridisations of atomic orbitals of nitrogen in NO2+,
NO–3 and NH+4 respectively are: (2016 - II) 29. Maximum bond angle at nitrogen is present in which of the
3
(a) sp, sp and sp 2 2 3
(b) sp , sp and sp following? (2015)

(c) sp, sp2 and sp3 (d) sp2, sp and sp3 (a) NO2+ (b) NO3– (c) NO2 (d) NO2–

21. Which of the following pairs of ions is isoelectronic and 30. Which of the following molecules has the maximum dipole
isostructural? (2016 - II)
moment? (2014)
(a) CH4 (b) NH3 (c) NF3 (d) CO2
(a) CO32– , NO3– (b) ClO3– , CO32–
(c) SO32– , NO3– (d) ClO3– , SO32– 31. Which one of the following species has planar triangular shape?
 (2014)
22. Consider the molecules CH4, NH3 and H2O. Which of the (a) NO3– (b) NO2–
given statement is false? (2016 - I)
(c) CO2 (d) N3
(a) The H─C─H bond angle in CH4 is larger than the
H─N─H bond angle in NH3 32. Which of the following organic compounds has same
hybridisation as its combustion product (CO2)? (2014)
(b) The H─C─H bond angle in CH4, the H─N─H bond (a) Ethyne (b) Ethene
angle in NH3, and the H─O─H bond angle in H2O are all
(c) Ethanol (d) Ethane
greater than 90º.
33. Which of the following is a polar molecule? (2013)
(c) Then H─O─H bond angle in H2O is larger than the
(a) BF3 (b) SF4
H─C─H bond angle in CH4
(c) SiF4 (d) XeF4
(d) The H─O─H bond angle in H2O is smaller than the
34. Which of the following is electron-deficient? (2013)
H─N─H bond angle in NH3
(a) (CH3)2 (b) (SiH3)2
23. Predict the correct order among the following: (2016 - I)
(c) (BH3)2 (d) PH3
(a) Lone pair ─ bond pair > bond pair ─ bond pair > lone
35. Which of the following is paramagnetic? (2013)
pair ─ lone pair
(b) Lone pair ─ lone pair > lone pair ─ bond pair > bond pair (a) CO (b) O −2 (c) CN– (d) NO+
─ bond pair 36. Four diatomic species are listed below. Identify the correct
(c) Lone pair ─ lone pair > bond pair ─ bond pair > lone pair order in which the bond order is increasing in them:(2012 Mains)
─ bond pair (a) He2+ < O2– < NO < C22 – (b) NO < O2– < C22 – < He2+
(d) Bond pair ─ bond pair > lone pair ─ bond pair > lone pair (c) O2– < NO < C22 – < He2+ (d) C22 – < He2+ < O2– < NO
─ lone pair

96 Dropper NEET
37. In the replacement reaction 39. Bond order of 1.5 is shown by:  (2012 Pre)
(a) O2 (b) O2+

(c) O 2 (d) O22 –
40. Which one of the following pairs is iso-structural (i.e., having
the same shape and hybridisation)?  (2012 Pre)
The reaction will be most favorable if M happens to be: – +
(a) [BF4 and NH4 ] (b) [BCl3 and BaCl3]
(2012 Mains)
(c) [NH3 and NO3–] (d) [NF3 and BF3]
(a) Li (b) Na (c) K (d) Rb
41. During change of O2 to O −2 ion, the electron adds on which one
38. The pair of species with the same bond order is: (2012 Pre)
of the following orbitals? (2012 Mains)
(a) N2 , O2 (b) O22 –, B2 (a) π* orbital (b) π orbital
+
(c) O2 , NO + (d) NO , CO (c) σ* orbital (d) σ orbital

Chemical Bonding and Molecular Structure 97


ANSWER KEY
Topicwise Questions

1. (c) 2. (b) 3. (b) 4. (b) 5. (d) 6. (c) 7. (b) 8. (b) 9. (b) 10. (d)
11. (d) 12. (c) 13. (d) 14. (b) 15. (c) 16. (b) 17. (c) 18. (d) 19. (a) 20. (c)
21. (a) 22. (a) 23. (d) 24. (b) 25. (c) 26. (c) 27. (a) 28. (d) 29. (c) 30. (c)
31. (b) 32. (b) 33. (a) 34. (b) 35. (b) 36. (b) 37. (c) 38. (d) 39. (b) 40. (c)
41. (c) 42. (c) 43. (a) 44. (b) 45. (a) 46. (a) 47. (c) 48. (a) 49. (b) 50. (b)
51. (a) 52. (b) 53. (d) 54. (c) 55. (a) 56. (d) 57. (c) 58. (d) 59. (d) 60. (a)
61. (d) 62. (b) 63. (b) 64. (d) 65. (d) 66. (c) 67. (c) 68. (d) 69. (d) 70. (c)
71. (b) 72. (b) 73. (d) 74. (b) 75. (c) 76. (b) 77. (c) 78. (b) 79. (a) 80. (b)
81. (c) 82. (d) 83. (c) 84. (b) 85. (b) 86. (c) 87. (c) 88. (a) 89. (b) 90. (b)
91. (a) 92. (d) 93. (a) 94. (b) 95. (c) 96. (b) 97. (b) 98. (b) 99. (d) 100. (b)
101. (a) 102. (c) 103. (c) 104. (b) 105. (d) 106. (c) 107. (d) 108. (a) 109. (a) 110. (c)
111. (a) 112. (d) 113. (d) 114. (d) 115. (b) 116. (a) 117. (b) 118. (b) 119. (b) 120. (a)
121. (c) 122. (d) 123. (a) 124. (a) 125. (a) 126. (b) 127. (d) 128. (a) 129. (c) 130. (b)
131. (d) 132. (c) 133. (d) 134. (d) 135. (c) 136. (d) 137. (c) 138. (b) 139. (b) 140. (b)
141. (c) 142. (b) 143. (a) 144. (b) 145. (c) 146. (c) 147. (a) 148. (a) 149. (c) 150. (b)
151. (b) 152. (c) 153. (c) 154. (a) 155. (a) 156. (b) 157. (a) 158. (b) 159. (b) 160. (c)
161. (b) 162. (c) 163. (d) 164. (b) 165. (b) 166. (a) 167. (d) 168. (d) 169. (a) 170. (a)
171. (b) 172. (a) 173. (b) 174. (d) 175. (a) 176. (a) 177. (c) 178. (b) 179. (a) 180. (b)
181. (c) 182. (d) 183. (b) 184. (d) 185. (c) 186. (d) 187. (d) 188. (c) 189. (a) 190. (a)
191. (d) 192. (c) 193. (b)

Learning Plus
1. (b) 2. (a) 3. (c) 4. (c) 5. (a) 6. (b) 7. (c) 8. (b) 9. (c) 10. (d)
11. (a) 12. (c) 13. (d) 14. (a) 15. (c) 16. (c) 17. (a) 18. (d) 19. (c) 20. (c)
21. (b) 22. (a) 23. (c) 24. (c) 25. (b) 26. (d) 27. (b) 28. (a) 29. (b) 30. (a)
31. (d) 32. (a) 33. (c) 34. (d) 35. (a) 36. (c)

Multiconcept MCQs
1. (d) 2. (c) 3. (d) 4. (a) 5. (c) 6. (c) 7. (d) 8. (b) 9. (c) 10. (c)
11. (c) 12. (a) 13. (b) 14. (c) 15. (a) 16. (b) 17. (a) 18. (d) 19. (b) 20. (c)
21. (d) 22. (b) 23. (a) 24. (d)

NEET Past 10 Years Questions


1. (b) 2. (d) 3. (b) 4. (c) 5. (d) 6. (b) 7. (d) 8. (d) 9. (c) 10. (d)
11. (b) 12. (d) 13. (a) 14. (d) 15. (c) 16. (d) 17. (a) 18. (c) 19. (a) 20. (c)
21. (a,d) 22. (c) 23. (b) 24. (b) 25. (a) 26. (a) 27. (c) 28. (b) 29. (a) 30. (b)
31. (a) 32. (a) 33. (b) 34. (c) 35. (b) 36. (a) 37. (d) 38. (b) 39. (c) 40. (a)
41. (a)

98 Dropper NEET

You might also like